XX International Physics Olympiad
\.80111A1
Ph
17‘N
/ S
Ofd
tiff
TeP
Ao
lava 16-24 311
"
Editor:
Waldemar ...
442 downloads
1597 Views
13MB Size
Report
This content was uploaded by our users and we assume good faith they have the permission to share this book. If you own the copyright to this book and it is wrongfully on our website, we offer a simple DMCA procedure to remove your content from our site. Start by pressing the button below!
Report copyright / DMCA form
XX International Physics Olympiad
\.80111A1
Ph
17‘N
/ S
Ofd
tiff
TeP
Ao
lava 16-24 311
"
Editor:
Waldemar Gorzkowski Institute of Physics Polish Academy of Sciences
%b World Scientific Singapore • New Jersey • London • Hong Kong
Published by World Scientific Publishing Co. Pte. Ltd. P 0 Box 128, Fairer Road, Singapore 9128 USA office: 687 Hartwell Street, Teaneck, NJ 07666 UK office: 73 Lynton Mead, Totteridge, London N20 8DH
XX INTERNATIONAL PHYSICS OLYMPIAD Copyright © 1990 by World Scientific Publishing Co. Pte. Ltd.
All rights reserved. This book, or parts thereof, may not be reproduced in any form or by any means, electronic or mechanical, including photocopying, recording or any information storage and retrieval system now known or to be invented, without written permission from the Publisher.
ISBN 981-02-0084-6
Printed in Singapore by JBW Printers & Binders Pte. Ltd.
To My Wife The Editor
vii
CONTENTS
1. The emblem
ix
2. Preface
xi
3. Organizing Committee
xvii
4. Sponsors (and co-organizers)
xxi
5. International Board, Observers, Guests 6. List of Competitors 7. Programme 8. Problems Problem 1 Problem 2 Problem 3 Experimental problem Histograms of marks 9. Minutes
xxvii xxxix xliii 1 1 11 23 32 43 46
10. UNESCO meeting
54
11. Prize - winners
57
12. Statutes
62
13. Syllabus
72
14. History and Perspectives
82
ix
THE EMBLEM OF THE XX INTERNATIONAL PHYSICS
OLYMPIAD
The emblem of the XX International Physics Olympiad contains a picture that is a record of the first hypernuclear event observed and interpreted in Warsaw by M. 1 Danysz and J. Pniewski.The collision of a high energy cosmic particle (labelled with "p" in the figure) with a heavy nucleus A was registered in nuclear emulsion. Tracks of the secondary particles emitted in the event seen in the picture consist of tracks due to fast pions (thin tracks) and to much slower fragments of the target nucleus ("black" tracks). The "black" track denoted with "h" in the figure is due to a hypernuclear fragment, in this case due to a part of the primary nucleus containing an unstable hyperon A instead of one of the nucleons. Hyperfragments are a new kind of matter in which the nuclei contain not only protons and electrons but also some more heavy particles. In the event observed above the hyperon A is bound with nucleons and decays like a free particle through a weak (slow) process only. This fact itself strongly suggested existence of a new quantum number which could explain suppression of fast decay. even in the presence of nucleons. Indeed, this was one of the observations that. 30 months later, led to the concept. of strangeness.
1
M. Danysz and J. Pniewski, Bull. AcVid. Po/.0n. Sci., 3(1). 42 (1952) and Phil.. Hag., 44, 348 (1953) y
xi
PREFACE
International Physics Olympiad is an annual competition in physics for secondary school students. First such contest took place in Warsaw in 1967. The present, twentieth International Physics Olympiad, is a jubilee event. Most details concerning the organization and course of the competition are to be found in the main text. The information given below, however, is not given elsewhere. Teams from the following countries were invited: 1. 2. 3. 4. 5. 6. 7. 8. 9. 10. 11. 12. 13. 14. 15. 16. 17. 18. 19. 20.
Albania Australia Austria Belgium Bulgaria Canada China Colombia Cuba Cyprus Czechoslovakia Denmark Federal Republic of Germany Finland France German Democratic Republic Great Britain Greece Hungary Iceland
21. 22. 23. 24. 25. 26. 27. 28. 29. 30. 31. 32. 33. 34. 35. 36. 37. 38. 39. 40.
Islamic Republic of Iran Italy Japan Korea (5) Kuwait Nepal The Netherlands Norway Poland Romania Singapore Soviet Union Spain Sweden Switzerland Turkey United Arab Emirates United States of America Vietnam Yugoslavia
xii
alongside with the observers from: 1. Kenya 3. Thailand 4. Zambia as well as observers from two international organizations: 1. The European Physical Society 2. UNESCO Teams from the following countries them with additional observers): 1. 2. 3. 4. 5. 6. 7. 8. 9. 10. 11. 12. 13. 14. 15.
Australia Austria Belgium Bulgaria Canada China Colombia Cuba Cyprus Czechoslovakia Federal Republic of Germany Finland German Democratic Republic Great Britain Hungary
16. 17. 18. 19. 20. 21. 22. 23. 24. 25. 26. 27. 28. 29.
arrived
(some
of
Iceland Islamic Republic of Iran Italy Kuwait The Netherlands Norway Poland Romania Singapore Soviet Union Sweden Turkey United States of America Yugoslavia
The following countries were represented by observers: 1. Greece 2. Spain 3. Thailand 4. United Arab Emirates
XIII
The observer from the European Physical Society was also present (UNESCO did not send its observer). Additionally, the Organizing Committee invited the team from Lithuania to unofficial participation in the competition. If the Lithuanian pupils had participated officially, their results would have won them one silver medal and one honorary mention: the medal: the mention:
Vincas Tamo§iunas Aidas Alaburda
(30.833 points) (26.833 points)
Because of the jubilee the Organizing Committee invited seven former members of the International Board, whose contribution for development and popularization of the International Physics Olympiads had been invaluable. Unfortunately, not all were able to attend. Even so, we had the pleasure of welcoming three of them, namely: Ms. Galina Sergeyevna Tarasyuk (Soviet Union) Prof. Dratko Grujid (Yugoslavia) Prof. Jaochim Wendt (German Democratic Republic) The presence of our guests-of-honour made for the stimulating atmosphere, and Prof. Wendt's recollective lecture illustrated with many slides, was particularly interesting for the more recent participants. Many thanks are due to the sponsors, whose financial support made the Olympiad at all possible. Particular mention must be made of the institutions involved jointly in the organizational effort (see the list of sponsors and co-organizers).
xiv The students and their supervisors were housed in two to Kuratorium 03wiaty dormitory houses belonging Wychowania in Warsaw: the delegation leaders: Internat Liceum Ogolnoksztalcacego im. LLP ul. Lindego 20, 01-952 Warszawa the students: Internat ZSZ nr. 5 ul. 2eromskiego 81A, 01-882 Warszawa Many thanks are given to the staff of both dormitories. The Organizing Committee attached greatest importance to the competition problems. Work on these had lasted for six months. All the problems of former competitions had been scrupulously analyzed, and all critical remarks considered. The aim of our activity had been to produce problems which would be interesting. requiring creativity, intellectually challenging, yet possible to solve within the confines of the secondary school physics. Care was taken to ensure that the problems do not put at disadvantage, by the content or wording, any of the participating teams. Nor did we wish to give unfair advantage to any other. It was not an easy task since the syllabuses and teaching traditions vary in different countries. It is for the Reader to decide whether or not the Organizing Committee has succeded. Again, thanks are due to the referees of the problems, especially to DSc. Andrzej Szymacha from the Warsaw University who worked with great personal dedication. The organization of the contest for 30 teams is not easy task. The enormity of organizational effort escapes description. Hence, speaking on behalf of the Organizational Committee (as well as my own) I would like to express our
xv
heartfelt gratitude to Dr. Andrzej Nadolny who had assumed the function of the Director and worked with exceptional sacrifice. There is no need to discuss in more detail the arrangement of the Proceedings, which is typical. One departure from tradition is the final chapter treating on the history of the International Physics Olympiads, which, it being a jubilee event, seems natural. The text of the experimental problem (including its solution) was prepared by Dr. Andrzej Kotlicki. the Author of this problem. The other parts of the book were prepared by the Editor, partly with the kind and disinterested help of his Friends: Mrs. Marzena Reich, MSc., and Mr. Andrzej Reich, MSc., to whom I give my warm thanks. The Editor
xvii
ORGANIZING COMMITTEE OF THE XX INTERNATIONAL PHYSICS OLYMPIAD
1.
President:
Prof. Grzegorz Bialkowski (Warsaw University)
Acting President:
Prof. Jan Blinowski (Warsaw University)
Vice-Presidents:
Prof. Jerzy Prochorow
Director: Members:
(Institute of Physics) Prof. Henryk Szymczak (Institute of Physics) Dr. Andrzej Nadolny (Institute of Physics) MSc. Witold Bober (Ministry of Education) Dr. Waldemar Gorzkowski (Institute of Physics) Dr. Andrzej Kotlicki (Warsaw University) MSc. Wladyslawa Piwofiska (Ministry of Education) Prof. Ewa Skrzypczak (Warsaw University) Prof. Stanislaw Woronowicz (Warsaw University)
1
Professor Grzegorz Bia}kowski, physicist famous for his papers on high energy physics, poet, President of the Warsaw
University, Senator-elect to the Polish Parliament, died suddenly on June 29, 1989, less than three weeks before the opening of the XX International Physics Olympiad.
XVIII
Subcommittee for the competition problems Co-heads:
Dr. Waldemar Gorzkowski (theory) Dr. Andrzej Kotlicki (experiment)
Problem. I
Author: Referees:
Dr. Waldemar Gorzkowski Dr. Andrzej Szadkowski DSc. Andrzej Szymacha Dr. Wlodzimierz Ungier
Marking team:
Dr. Waldemar Gorzkowski (head) Dr. Andrzej Szadkowski Dr. Wlodzimierz Ungier
Problem 2
Author:
Dr. Waldemar Gorzkowski
Referees:
Dr. Andrzej Szadkowski DSc. Andrzej Szymacha Dr. Wlodzimierz Ungier Prof. Stanislaw Woronowicz
Marking team:
DSc. Andrzej Szymacha (head) Dr. Piotr Kielanowski Dr. Michal Spalinski
Problem 3
Author: Referees:
Dr. Waldemar Gorzkowski Dr. Andrzej Raica DSc. Andrzej Szymacha
xix Dr. Wiodzimierz Ungier Dr. Andrzej Rajca (head) Dr. Krzysztof Meissner
Marking team:
Dr. Wojciech Szuszkiewicz Experimental problem
Author:
Dr. Andrzej Kotlicki
Referees:
MSc. Krzysztof Korona MSc. Anna Lipniacka MSc. Jerzy Lusakowski Dr. Bruno Sikora
Marking team:
Dr.' Andrzej Kotlicki (head) Dr. Aleksy Bartnik Dr. Krzysztof Genser Dr. Jedrzej Jedrzejewski MSc. Krzysztof. Korona MSc. Jerzy Lusakowski Dr. Joanna Schiller Dr. Bruno Sikora
Permanent Co workers of the Organizing Committee -
Secretary:
Ms. Beata Harazinska Head
of the group of interpreters:
Dr. Danuta Kurzyca (Warsaw University) Accommodation and transportation:
XX
MSc. Jacek Goszczydski Ms. Hanna Kapu..Iniak Ms. Stanistawa Turczydska Excursions and cutturaL program:
MSc. Ewe Wieckowska Accountant:
Ms. Halina Kielek Cashier:
Ms. Maigorzata Imiiiska Computer center:
Dr. Marek Gutowski Decoration and design:
MSc. Dariusz Cacek Provider:
Mr. Teodor Jarosidski
xxi
SPONSORS OF THE XX INTERNATIONAL PHYSICS
OLYMPIAD
Ministerstwo Edukacji Narodowej, Warszawa (main sponsor and co-organizer) Instytut Fizyki Polskiej Akademii Nauk, Warszawa (sponsor and co-organizer) Instytut Fizyki Dotwiadczalnej Uniwersytetu Warszawskiego, Warszawa (sponsor and co-organizer) Polskie Towarzystwo Fizyczne, Warszawa (co-organizer) Instytut Fizyki Teoretycznej Uniwersytetu Warszawskiego, Warszawa Fundacja im. Stefana Batorego, Warszawa Uniwersytet Warszawski, Warszawa Komitet do spraw Miodziety i Kultury Fizycznej, Warszawa Urzad Postepu Naukowo-Technicznego i Wdro±ell, Warszawa Ministerstwo Przemyslu, Warszawa Centralny Otrodek Badawczo-Rozwojowy Aparatury Badawczej i Dotwiadczalnej, Warszawa Wydziai Fizyki Technicznej i Matematyki Stosowanej Politechniki Warszawskiej, Warszawa
xxii
Wydziai Elektroniki Politechniki Warszawskiej, Warszawa Centrum Badan Kosmicznych Polskiei Akademii Nauk, Warszawa Instytut ProblemOw Jqdrowych, Otwock-Swierk Zakiad Dotwiadczalny Aparatury Jadrowej, Otwock-Swierk Cynel-Unipress, Sp. z o.o., Warszawa Przedsiebiorstwo Poloniino-Zagraniczne "Slandi", Opacz-Michalowice Krajowy Fundusz na Rzecz Dzieci, Warszawa Instytut Fizyki Jadrowej, Krakdiw Zjednoczone Zaklady Urzkdzen Jkdrowych "POLON", Warszawa Instytut Niskich Temperatur i Bade' Strukturalnych Polskiei Akademii Nauk, Wroclaw Laboratorium Wzrostu KrysztalOw, Sp. z o.o., Warszawa Instytut Technologii Elektronowej, Warszawa Polskie Zaklady Optyczne, Warszawa Zaklady Elektromaszynowe "CELMA", Cieszyn Warszawskie Zaklady Kaletnicze "NOMA", Warszawa Kombinat Przemyslu Narzedziowego "VIS", Warszawa PrzedsiQbiorstwo Zagraniczne "AMEPOL", Warszawa
XXIII
Wydzial Matematyki i Fizyki Uniwersytetu Jagielionskiego, KrakOw Wojskowa Akademia Techniczna, Warszawa Przemyslowy Instytut Automatyki i PomiarOw "MERA-PIAP", Warszawa Przemysiowy Instytut Elektroniki, Warszawa Zakiady WI6kien Chemicznych Chemitex-Wistom, TomaszOw Mazowiecki Fabryka SamochodOw Osobowych, Warszawa Przedsivbiorstwo PoloniJno-Zagraniczne "ALEXIS", E6d2 PrzedsiQbiorstwo Realizacji ObiektOw Energetycznych i Przemystowych "MEGAT-MEGADEX", Warszawa Zaklad Wysokich Cienien "UNIPRESS", Warszawa Lubuskie Zaklady Aparatew Elektrycznych "LUMEL", Zielona GOra Instytut Chemii Fizycznej Polskiej Akademii Nauk, Warszawa Dom Handlowy Nauki Polskiej Akademii Nauk, Warszawa Polskie Biuro Podraty "Orbis",
Warszawa
WojewOdzki 06rodek Politechniczny, Warszawa Zesp61 Szkol Elektronicznych im. PPR, Warszawa
xxiv Zaklady Radiowe im. Marcina Kasprzaka, Warszawa Centralny 0rodek Badawczo-Rozwojowy "POLAM", Warszawa Prof. Iwo Bialynicki-Birula, Warszawa Zwikzek Rekodziela Ludowego i Artystycznego "Cepelia", Warszawa Panstwowe Wydawnictwo Naukowe, Warszawa Warszawskie Zaklady Urzkdzen Informatyki "Meramat",Warszawa Krajowa Agencja Wydawnicza, Warszawa Wydawnictwo "ARKADY", Warszawa Wydawnictwa Artystyczne i Filmowe, Warszawa Wydawnictwo "Sport i Turystyka", Warszawa Instytut Fizyki Molekularnej Polskiej Akademii Nauk, Poznan Centralna Skladnica Harcerska, Warszawa Stowarzyszenie In2ynierOw i TechnikOw Rolnictwa, Warszawa Mazowieckie Zaklady Rafineryjno-Petrochemiczne, Flock Miejskie Zaklady Komunikacyjne, Warszawa Miodzie2owa Agencja Wydawnicza, Warszawa SpOldzielczy Zakiad Ubezpieczen "WESTA", Warszawa
xxv European Physical Society, Geneve World Scientific Publishing Company, Singapore
xxvii INTERNATIONAL 1989/1990
BOARD
President: Professor Jan Blinowski Institute of Theoretical Physics Warsaw University ul. HoZa 69 00-681 Warszawa Poland Members: AUSTRALIA
Prof. Rodney L. Jory Department of Physics Faculty of Science Australian National University GPO Box 4 Canberra ACT 2601
Dr. Todor Petrov Department of Physics Faculty of Science Australian National University GPO Box 4 Canberra ACT 2601
AUSTRIA
Prof. Ing. Mag. Helmuth Mayr BRG 15 Auf der Schmelz 4 A - 1150 Wien
Prof. Mag. GUnther Lechner Bundesrealgymnasium Wargl InnsbruckerstraBe 34 WOrgl A - 6300 WOrgl/Tirol
XXVIII
BELGIUM
Prof. Jacques Keil Heidhohe 1 B - 4700 Eupen
Prof. Marc Beddegenoodts Mollenveldwijk, 30 B - 3280 Zichem
BULGARIA
Dr. Maxim Maximov Fizicheskiy Facultet Blvd. Anton Ivanov 5 BG - 1126 Sofia
Ms. Elena Genova Ministerstvo Narodnogo Prosveshcheniya Blvd. Stambolijski 18 Sofia
CANADA
Dr. John Wylie Canadian Chemistry and Physics Olympiad 306 Lawrence Ave. E. Toronto, Ontario M4N 1T7
Dr. Michael Crooks Department of Physics University of British Columbia Vancouver B. C. V6T 1W5
CHINA
Prof. Zhu Shi Gia Department of Physics University of Beijing Beijing
Prof. Shu You Sheng Department of Physics University of Beijing Beijing
COLOMBIA
Prof. Fernando Vega Salamanca Cl 58A N. 37-70 Bogota D. E.
Prof. Luis Aleiandro Ladino Gaspar Cl 58A N. 37-70 Bogota D. E.
CUBA
Prof. Carlos Enrique Sifredo Barrios Ministerio de Educacion Obispo 160 e/Mercaderes y San Ignacio Habana
Prof. Raul Antonio Portuondo Duany Facultad de Fisica Universidad de la Habana San Lazoro y L. Habana
CYPRUS
Dr. Constantinos Pougourides Mr. Panayiotis Christodoulou P.O.Box 4863 28 Olymbias Str. Lykavittos Nicosia Nicosia CZECHOSLOVAKIA
Dr. Daniel Kluvanec Pedagogicka Fakulta Katedra Fyziky Lomonosova 1 CS - 94974 Nitra
Dr. Vaclav Sula Ministerstvo gkolstvi, Mlade2 a Televychovy Karmelitska 7 CS - 11812 Praha 1
xxx FEDERAL REPUBLIC OF GERMANY Dr. Gunter Lind IPN, Universitat Kiel Olshausenstr. 62 D - 2300 Kiel
Dr. Harri Heise Sickendamm 26 D - 2240 Heide
FINLAND Dr. Maija Ahtee Department of Teacher Education University of Helsinki Ratakatu 2 SF - 00120 Helsinki
Mr. Jukka Mattila Isoniityntie 1 SF - 21600 Parainen
GERMAN DEMOCRATIC REPUBLIC Dr. Rudolf Gau Padagogische Hochschule "Liselotte Herrmann" Goldberger Str. 12 DDR - 2600 GUstrow
Dr. Christoph Schick Padagogische Hochschule "Liselotte Herrmann" Goldberger Str. 12 DDR - 2600 GUstrow
GREAT BRITAIN Dr. Cyril Isenberg Physics Laboratory University of Kent Canterbury Kent CT2 7NR
Mr. Guy Bagnall 2, Kennet House Harrow Park Harrow-on-the-Hill MIDDX. HA1 3JE
HUNGARY
Dr. JenO Szep Roland EbtvOs University Muzeum krt. 6-8 H - 1088 Budapest
Dr. Peter Gnadig Roland EOtviis University Puskin u. 5-7 H - 1088 Budapest
ICELAND
Dr. Einar JtaliUsson University of Iceland Taeknigarour Dunhaga 5 IS - 107 Reykjavik
Mr. Vidar AqUstsson SKYRR IS - 108 Reykjavik
ISLAMIC REPUBLIC OF IRAN
Dr. Mohammad Sepehry Rad Physics Department Shahidbeheshty University Tehran - 19834
Dr. Ahmad Shirzad Physics Department Sharif University of Technology Tehran
ITALY
Prof. Paolo Violino Dipartimento di Fisica Sperimentale Universita di Torino V. P. Giuria 1 I - 10125 Torino
Prof. Giuliana Cavaggioni Via Paliaga 3/2 I - 30030 Venezia Tessera
KUWAIT
Mr. Jaber M. Shaqalaih Science Supervision Department Ministry of Education P.O.Box 7, Safat 13001
Mrs. Rahma M. Zaqout Science Supervision Department Ministry of Education P.O.Box 7, Safat 13001
THE NETHERLANDS Dr. Hans Jordens Rijksuniversiteit Groningen Rozengaard 5 NL - 9753 BK Haren
Dr. Anne Holvast Rijksuniversiteit Utrecht Lijsterstraat 17 NL - TA Utrecht
NORWAY Ms. Ingerid Hiis Helstrup Langhaugen Skole Hagerups vej 17 N - 5030 Landaas
Mr. Svein Lie Ovenbakken 16 A N - 1345 Oesteraas
POLAND Dr. Miroslaw Hamera Institute of Physics Polish Academy of Sciences al. Lotnik6w 32/46 PL - 02 668 Warszawa
Dr. Jan Mostowski Institute of Physics Polish Academy of Sciences al. Lotnikow 32/46 PL - 02 668 Warszawa
ROMANIA
Prof. Dan Iordache Physics Department Polytechnic Institute of Bucharest Splaiul Independentei Nr. 313 Bucharest
Prof. Romulus Pop Liceul Gh. Sincai Str. 17 Octombrie Nr. 25 Baia Mare 4800
SINGAPORE
Prof. Phee Poh Ong Physics Department National University of Singapore Kent Ridge Singapore 0511
Mr. Chee Hau Gan Sciences Branch Ministry of Education Kay Siang Road Singapore 1024
SWEDEN
Dr. Lars Gislen Dr. Lennart Johansson Dept. of Theoretical Physics Physics Department Solvegatan 14 A SOlvegatan 14 B S - 223-62 Lund S - 223--62 Lund SOVIET UNION
Dr. Sergey S. Krotov Dr. Moscow State University NII Department of Physics ul. SU - 117234 Leninskiye Gory SU -
Vladimir A. Orlov SIMO APN Kosmonavtov 18, korpus 1 129243 Moskva
xxxiv TURKEY
Dr. Ibrahim GUnal Department of Physics METU TR - 06531 Ankara
Prof. Ordal Demokan Department of Physics METU TR - 06531 Ankara UNITED STATES
Dr. Larry Kirkpatrick Montana State University Department of Physics Bozeman, MT 59717
Dr. Arthur Eisenkraft 60 Stormytown Rd. Ossining, NY 10562
YUGOSLAVIA
MSc. Kreo Zadro Department of Physics Faculty of Science P.O.Box 162 YU - 41000 Zagreb
MSc. Ana Smontara Institute of Physics of the University Bijenicka 46, P.O.Box 304 YU - 41000 Zagreb
HEADS
OF
THE
LITHUANIAN
TEAM
(Unofficial participation in the XX IPhO) Prof. Antanas-Rimvidas Bandzaitis Institute of Physics Lithuanian Academy of Sciences ul. K. Pozhelos 54 Vilnius Lithuania - USSR
Mr. Pavel 0. Bogdanovich Institute of Physics Lithuanian Academy of Sciences ul. K. Pozhelos 54 Vilnius Lithuania -- USSR
XXXV
GUESTS
OF
HONOUR
1. Ms. Galina Sergeyevna Tarasyuk ul. Profsoyuznaya 42-3-69 SU - 117335 Moskva Soviet Union 2. Prof. Joachim Wendt Padagogische Hochschule "Liselotte Herrmann" Goldberger Str. 12 DDR - 2600 GUstrow German Democratic Republic 3. Prof. Drako Grujid ul. Gospodar Jevremova 16 YU - 11000 Beograd Yugoslavia
OBSERVERS from new countries: GREECE
Dr. Nick Vassilopoulos Evdoxou 8-10 GR - 11743 Athens
Dr. Athanassios Faloukas Olvbou 56 GR - 15234 Halapari
SPAIN
Prof. Antonio Bernalte Miralles Universidad N. de Educacion a Distancia Facultad de Ciencias Apartado 60141 E - 28080 Madrid
xxxvi THAILAND
Prof. Kongkan Bhatrakarn Physics Department Faculty of Science Kasetsart University Bangkok 10900
Prof. Chaleo Manilerd Director of the Institute for the Promotion of Teaching Science and Technology 924 Sukhumvit Road Bangkok 10110
UNITED ARAB EMIRATES
Mrs. Hamda Yousif Lootah P.O.Box 147 Dubai
Mr. Jamal Khalifa Lootah P.O.Box 147 Dubai
from the European Physical Society: Prof. Joseph Depireux Institute of Physics University of Liege Sart Tilman B - 4000 Liege 1 Belgium from "old" countries: AUSTRALIA
Mr. James Lloyd c/o Mr. Peter Lloyd Australian Embassy to Iraq Bagdad Iraq
xxxvii BELGIUM
Mr. Vincent Bruneau Teacher College Basse-Wavre Avenue des Sapins, 20 B - 5982 Biez
Mr. Lucien Beerden Teacher-Assistant Limburgs Universitair Centrum Stevoortse Kiesel, 219 B - 3512 Hasselt
CHINA
Mr. Huang Yu Min High Education Department of the State Educational Committee Beijing GREAT BRITAIN
Mr. Conrad McDonell Merton College Oxford OX1 4JD HUNGARY
Dr. Erzsebet Lugosi Bukarest street 17 H - 1114 Budapest ITALY
Mr. Francesco Minosso Via S. Fermo 6 I - 30174 Venezia-Mestre
XXXVIII
THE NETHERLANDS Ms. Hennis Deenen contact via the delegation leaders SINGAPORE Mr. Willie S. M. Yong AMK Avenue 10 Apt. Blk 551 # 14-2224 Singapore 2056 YUGOSLAVIA Mr. Labud Vuk6evid Faculty of Physics and Mathematics YU - 81000 Titograd
ACCOMPANYING
PERSONS
Mrs. J. M. Bagnall (Great Britain) Ms. Ariya Bhatrakarn (Thailand) Mrs. Maria Falouka (Greece) Mr. Romano Righi (Italy) Mrs. Chrysanti Tzangauri (Greece) Mrs. Giustina Violino Marenco (Italy) Mr. Marco Violino (Italy) Mrs. A. Wendt (GDR)
LIST
OF
COMPETITORS
AUSTRALIA
AUSTRIA
Brett Munro Gareth Williams Arthur Street Clement Loy Mattew Brecknell
Gregor Weihs Hannes Sakulin ainther Lang Norbert Schbrghofer Thomas Bednar
BELGIUM
BULGARIA
Gregory Lielens Pascal Pirotte Patrick Dupont Cristophe Colle Miguel Vermandel
Asen Kumanov Stefan Piperov Konstantin Stefanov Yavor Velchev Dragomir Nechev
CANADA
CNINA
Eric Nodwell Chris Simons Nima Arkani-Hamed Jon-Paul Voroney Bradley Heinrichs
Ge Ning Lin Xiao Fan Qiu Dong Yu Mao Yong
COLUMBIA
CUBA
Roberto Hernando Vargas Cruz Gerardo A. MUNoz Quiriones Dario Fernando Martinez Mantilla
Leopoldo Avelino Pando Zayas Victor Lopez Richard Eduardo Ariel Menendez Proupin Alberto Serra Roure Elder Puppo Escalona
Yen Jing
xl CYPRUS
CZECHOSLOVAKIA
Georgios Ioannou Nicolaos Hadiiconstantonou Antonios Eleftheriou Charalambdos Anastasiou Adamos Papantoniou
Arnot Kobylka Petr Duczynski Andrei Dobot Zbynek Vaata Miroslav Vicher
FINLAND
FRG
Pekka Heino Janne Karimaki Otso Ovaskainen Timo Tarhasaari Timo Rantalainen
Volker Gebhardt Jens Lang Udo Karthaus Olaf Kummer Volker Springel
GDR
GREAT BRITAIN
Carsten Deus Andre Fraenzel Werner Torsten Thomas Wilcke Swen Wunderlich
Gregory Colyer Colin Merryweather Gordon Ogilvie Michael Rutter Richard Wilson
HUNGARY
ICELAND
Gabor Felso
Asta K. SveinsdOttir AgOst Vaifells Gunnar Palsson Halldor Poisson Kristian Leosson
Zoltan Hidv-egi Antal Karolyi Szabolcs Kesmarki Szilard Szabo
xli
ISLAMIC REPUBLIC OF IRAN
ITALY
Vahid Borumand Sani Human Dejnabadi Ramin Farjad Had Ramin Golestanian Mohammad Mehdi Khalighi
Paolo Azzuri Ciro Cattuto Silvano de Franceschi Massimiliano Guzzo Federico Toschi
KUWAIT
THE NETHERLANDS
Abdul Wahab J. Al Tourah Hassan N. Al Qattan Muna M. Al Dousari Wessam M. All Fatema A. Al Mohailij
Eric Cantor Romke Jonker Arnold Metselaar Martijn Mulders Rolf Oldeman
NORWAY
POLAND
Hans Olav Sundfer Rune Hasvold Oyvind Tafjord Fredrik Kvamme Roger Klausen
Piotr Kossacki Cezary Sliwa Tomasz Motylewski Romuald Jani.k Leszek Mencnarowski
ROMANIA
SINGAPORE
Gabriel Balan Mona Berciu Lucian Ciobica Viorel-Cristian Negoita Costin-Radu. Popescu
Desmond Rodney Lim Chin Siong Chua Soon Ghee Lim Shiang Liang Cheong Kwok Leong Tay Soo Tong
xlii
SOVIET UNION
SWEDEN
Konstantin Zuev Alecsandr Korshcov Nicolay Kuzma Vladislav Makeev Juru Uvarov
Simon Ekstrbm Mika NystrOm Jahan Axnas Stefan Jakobsson. Stefan Davidsson
TURKEY
USA
Mehmet Emin Alpay Emre Sermutlu Sencer Taneri Cem Odaman Huseyin Altura
Derrick Bass James Sarvis Jason Jacobs Jessica Millar Steven Gubser
YUGOSLAVIA
Zvonimir Bandid Viekoslav Mladineo Dubravko Tomasovid Dalibor Tu2inski Andrei Vilfan LITHUANIA - USSR Cunofficial participatioa)
Aidas Alaburda Marius Asipauskas Valdas Kira Dovydas Razas Vincas Tamo§iunas
PROGRAMME OF THE XX INTERNATIONAL PHYSICS
OLYMPIAD
Delegation Leaders ******* ***********
Saturday, July 16, 1989 Arrivals 13.30 - 14.30 18.00 - 20.00
Lunch Dinner
Monday, July 17, 1989 8.00 9.15 10.00
11.30 13.00 15.00 18.30 19.15 19.45
Breakfast Departure of busses for the Warsaw University Opening Ceremony of the XX International Physics Olympiad (Auditorium Maximum of the Warsaw University) Welcoming cocktail (Golden Chamber of the Casimir Palace) Lunch Meeting of the International Board: discussion on the theoretical problems Dinner Departure for the Institute of Physics, Polish Academy of Sciences Continuation of the meeting: translation and typing of the problems
xliv
Tuesday, July 18, 1989 8.30 10.00
13.15 14.30
18.00 18.30 19.30 20.00
Breakfast A meeting with representatives of the Polish educational system at the Ministry of National Education Lunch Departure for the Royal Castle; tour of the Royal Castle; walk around the Old Town Return to the boarding school Dinner Departure for the St. John's Cathedral Organ concert
Wednesday, July 19, 1989 7.45 8.30 11.00 12.45 13.00 14.00 16.30
18.30 19.15 19.45
Breakfast Departure for 2elazowa Wola Departure for Nieborow Departure for Lowicz Lunch in Lowicz ("Zajazd Lowicki" restaurant) Return to Warsaw Meeting of the International Board: discussion on the experimental problem(s) Dinner Departure for the Institute of Physics. Polish Academy of Sciences Continuation of the meeting: translation and typing of the problem(s)
xlv Thursday, July 20, 1989 8.30 9.15 10.00 13.15 15.15 16.30
19.45
Breakfast Departure for Wilan6w Tour of the Palace and Park in WilanOw Lunch in "Wilanowska" restaurant Return to the boarding school Meeting of the International Board: discussion on organizational problems of the International Physics Olympiads Dinner, with the students (Academy of Physical Education)
Friday, July 21, 1989 8.00 9.00
Breakfast Warsaw sightseeing tour
13.30 14.30
Lunch Discussion on the graded papers: group Al)- theoretical problems Dinner Lecture by Prof. Joachim Wendt (GDR) on previous International Physics Olympiads (illustrated with slides)
18.30 20.00
Saturday, July 22, 1989 8.00 9.15
Breakfast Discussion on the graded papers: group A - experimental problems, group B theoretical problems
13.30
Lunch
1)
All the countries participating in the competition were
divided into two groups: A and B.
xlvi
14.30 18.30 19.30
Discussion on graded papers: group B experimental problem(s) Dinner Final meeting of the International Board
Sunday, July 23, 1989 8.00 9.15
13.30 14.00 15.15 16.00
19.00
Breakfast Departure of busses for the centre of the city - free time for all the participants Return to the boarding school Lunch Departure for the Warsaw University Closing Ceremony of the XX International Physics Olympiad. including presentation of the awards (Auditorium Maximum of the Warsaw University) Dinner, with the students (Academy of Physical Education)
Monday, July 24, 1989 6.30 - 8.30
Breakfast Departures
xlvii
Students
* ******* The two times, separated by slash, refer to the group A and B respectively. Thus, for example, 7.45/8.30 means 7.45 for the group A and 8.30 for the group B.
Sunday, July 16, 1989 Arrivals 13.30 - 14.30 18.00 - 20.00
Lunch Dinner
Monday, July 17, 1989 7.30/8.15 9.15 10.00
13.00 14.30 18.30/19.15
Breakfast Departure of buses for the Warsaw University Opening Ceremony of the XX International Physics Olympiad (Auditorium Maximum of the Warsaw University) Lunch Warsaw sightseeing tour Dinner
Tuesday, July 18, 1989 7.15/8.00 9.00 14.00/14.45 14.45/15.15
Breakfast Competition: theoretical part Lunch Departure for the Royal Castle; tour of the Royal Castle; walk around the Old Town
xlviii
17.45 18.15/19.00 19.30 20.00
Return to the boarding school Dinner Departure for the St. John's Cathedral Organ concert
Wednesday, July 19, 1989 GROUP A
Breakfast Departure for Nieborow Departure for Arkadia Departure for 2elazowa Wola Departure for Lowicz Lunch in Lowicz ("Polonia" restaurant) Performance of a folk dancing group Return to Warsaw Dinner
7.15 8.00 11.00 11.45 13.30 14.45 15.30 16.45 18.30 GROUP B
8.00 8.30 11.00 12.15 14.00 15.30 17.00 19.00
Breakfast Departure for 2elazowa Wola Departure for Arkadia Departure for Niebordw Lunch in Nieborow ("Na Rozdro2u" restaurant) Performance of a folk dancing group Return to Warsaw Dinner
Thursday, July 20, 1989 GROUP A
7.00 7.45 8.30 14.15 14.45
Breakfast Departure for the physics laboratory Competition: experimental part Lunch Departure for the "Lazienki" Park; walk around the "Lazienki" Park; sightseeing Return to the boarding school Dinner, with the leaders (Academy of Physical Education)
18.30 19.45
GROUP B
7.45 9.00 11.30 12.30 13.15 14.00 19.45
Breakfast Departure for the "Lazienki" Park; walk around the "Lazienki" Park; sightseeing Return to the boarding school Lunch Departure for the physics laboratory Competition: experimental part Dinner, with the leaders (Academy of Physical Education)
Friday, July 21, 1989 7.15/7.45 8.00/8.15
Breakfast Departure for Olsztyn (2-day excursion)
Saturday, July 22, 1989 Return to Warsaw
20.00
Dinner
Sunday, July 23, 1989 7.45/8.30 9.15
13.30 14.00 15.15 16.00
19.00
Breakfast Departure of busses for the centre of the city - free time for all the participants Return to the boarding school Lunch Departure for the Warsaw University Closing Ceremony of the XX International Physics Olympiad, including presentation of the awards (Auditorium Maximum of the Warsaw University) Dinner, with the leaders (Academy of Physical Education)
Monday, July 24, 1989 6.30 - 8.30
Breakfast Departures
XX International Physics Olympiad
1
PROBLEMS OF THE XX INTERNATIONAL PHYSICS
OLYMPIAD
Problem 1
Consider two liquids A and B insoluble in each other. The pressures p (i = A or B) of their saturated vapours i obey, to a good approximation, the formula: a, In (p./po )
b •
(i = A or B)
T
where podenotes the normal atmospheric pressure, T the absolute temperature of the vapour, and ai and bi (i = A or B) - certain constants depending on the liquid. (The symbol In denotes the natural logarithm, i.e. logarithm with base e - 2.7182818...). -
The values of the ratio pi/pofor the liquids A and B at the temperatures 40°C and 90°C are given in Tab. 1.1.
Tab. 1.1
Pi/Po t [°C] i = A
i = B
40
0.284
0.07278
90
1.476
0.6918
The errors of these values are negligible. A) Determine the boiling temperatures of the liquids A
2 and B under the pressure pc,. B) The liquids A and B were poured into a vessel in which the layers shown in Fig. 1.1 were formed. The surface of the liquid B has been covered with a thin layer of a non-volatile liquid C, which is insoluble in the liquids A and B and vice versa, thereby preventing any free The evaporation from the upper surface of the liquid B. ratio of the molecular masses of the liquids A and B (in the gaseous phases) is:
1
=M A//jB = 8.
The masses of the liquids A and B were initially the same, each equal to m = 100 g. The heights of the layers of the liquids in the vessel and the densities of the liquids are small enough to make the assumption that the pressure in any point in the vessel is practically equal to the normal atmospheric pressure po. The system of liquids in the vessel is slowly, but continuously and uniformly, heated. It was established that the temperature t of the liquids changed with the time T as shown schematically in the Fig. 1.2. Determine the temperatures t 1and t2corresponding to the horizontal parts of the diagram and the masses of the liquids A and B at the time T i. The temperatures L i and t 2 should be rounded to the nearest degree (in °C) and the masses of the liquids should be determined to one-tenth of a gram.
REMARK: Assume that the vapours of the liquids, to a good approximation, (1)obey the Dalton law stating that the pressure of a mixture of gases is equal to the sum of the partial pressures of the gases forming the mixture, and (2)can be treated as perfect gases up to the pressures
3 corresponding to the saturated vapours. Po
Po
Po
Fig. 1.1
=
= = = = = = = = == = ==== = = = =2== = = =
=
t
t
2
Fig. 1.2 t
1
T 1
Solution.
PartA The liquid boils when the pressure of its saturated vapour is equal to the external pressure. Thus, in order to find the boiling temperature of the liquid i (i - A or B), one should determine such a temperature T bi (or tbi), for which pi/po = 1. Then In (p /p ) = 0, and we have: i
o
4 a. b.'
132
1
Thecoefficientsa.andb.are not given explicitly. However, they can be calculated from the formula given in the text of the problem. For this purpose one should make use of the numerical data given in the Tab. 1.1. For the liquid A, we have:
In 0.284 -
aA + b (40 + 273.15) K A
aA In 1.476 - (90 + 273.15) K + b A
After subtraction of these equations, we get: 1 In 0.284 - In 1.476 = aA 140 + 273.15 0.284 In 1.476 1 40 + 273.15
aA
1
1 90 + 273.15 )-K-1
K x -3748.49 K.
90 + 273.15
Hence,
b
A
= In 0.284
aA
(40 + 273.15) K
10.711.
Thus, the boiling temperature of the liquid A is equal to
T
bA = 3748.49K/10.711 x 349.95 K.
In the Celsius scale the boiling temperature of the liquid A is
5
t bA
= (349.95 - 273.15)°C = 76.80°C A-. 77°C.
For the liquid B, in the same way, we obtain:
a B
-5121.64 K,
bB A, - 13.735,
T
t
bB
bB
372.89 K,
99.74°C A: 100°C.
Part B
As the liquids are in thermal contact with each other, their temperatures increase in time in the same way. At the beginning of the heating, what corresponds to the left sloped part of the diagram, no evaporation can occur. The free evaporation from the upper surface of the liquid B cannot occur - it is impossible due to the layer of the non-volatile liquid C. The evaporation from the inside of the system is considered below. Let us consider a bubble formed in the liquid A or in the liquid B or on the surface which separates these liquids. Such a bubble can be formed due to fluctuations or for many other reasons, which will not be analyzed here. The bubble can get out of the system only when the pressure inside it equals to the external pressure p0 (or when it is a little bit higher than po). Otherwise, the bubble will collapse.
6 The pressure inside the bubble formed in the volume of the liquid A or in the volume of the liquid B equals to the pressure of the saturated vapour of the liquid A or B, respectively. However, the pressure inside the bubble formed on the surface separating the liquids A and B is equal to the sum of the pressures of the saturated vapours of both these liquids, as then the bubble is in a contact with the liquids A and B at the same time. In the case considered the pressure inside the bubble is greater than the pressures of the saturated vapours of each of the liquids A and B (at the same temperature). Therefore, when the system is heated, the pressure po is reached first in the bubbles which were formed on the surface separating the liquids. Thus, the temperature t i corresponds to a kind of common boiling of both liquids which occurs in the region of their direct contact. The temperature t1 is for sure lower than the boiling temperatures of the liquids A and B as then the pressures of the saturated vapours of the liquids A and B are less then p (their sum equals to pc), and each of them is greater than o zero). In order to determine the value of t 1 with required accuracy, we can calculate the values of the sum of the saturated vapours of the liquids A and B for several values of the temperature t and look when one gets the value po. From the formula given in the text of the problem, we have: A .„ T -A PA
T; =
(1)
e
B PB
7; p
A
+ p
B
equals to po , if
T e
-B (2)
7 P
7-A +PB 7- - 1. Po /-0 Thus, we have to calculate the values of the following function: a a B A + b + b t + t + t A B o o y(t) = e t = 273.15°C + e
t
o
and to determine the temperature t = t1, at which yCt, equals to 1. When calculating the values of the function y(t) we can divide the intervals of the temperatures t by 2 (approximately) and look whether the results are greater or less than 1. We have: Tab. 1.2 t
yCt,
40°C 77°C 59°C 70°C 66°C 67°C 66.5°C
< 1 (see the Tab. 1.1) > 1 (as t is less than t ) 1 bA 0.749 < 1 1.113 > 1 0.966 < 1 1.001 > 1 0.983 < 1
Thus, t 67°C (with required accuracy). 1 Now we calculate the pressures of the saturated vapours °C, i.e. of the liquids A and B at the temperature t i = 67 the pressures of the saturated vapours of the liquids A and B in each bubble formed on the surface separating the liquids. From the equations (1) and (2), we get:
8 pA x 0.734 po,
pB
0.267 po,
(PA + PB= 1.001 po x po). These pressures depend only on the temperature and, therefore, they remain constant during the motion of the bubbles through the liquid B. The volume of the bubbles during this motion also cannot be changed without violation of the relation pA + pB z po. It follows from the above remarks that the mass ratio of the saturated vapours of the liquids A and B in each bubble is the same. This conclusion remains valid as long as both liquids are in the system. After total evaporation of one of the liquids the temperature of the system will increase again (second sloped part of the diagram). However, the mass of the system remains constant until the temperature reaches the value t 2, at which the boiling of the liquid (remained in the vessel) starts. Therefore, the temperature t (the higher horizontal 2 part of the diagram) corresponds to the boiling of the liquid remained in the vessel. The mass ratio mA/mB of the saturated vapours of the liquids A and B in each bubble leaving the system at the temperature t1is equal to the ratio of the densities of these vapours eA/eB. According to the assumption 2, stating that the vapours can be treated as ideal gases, the last ratio equals to the ratio of the products of the pressures of the saturated vapours by the molecular masses:
9 " IAeA = PAPA - PA r• = m B eB P BP B PB Thus, m, -= x 22.0. " 1B We see that the liquid A evaporates 22 times faster than the liquid B. The evaporation of 100 g of the liquid A during the 'surface boiling' at the temperature t1 is associated with the evaporation of 100 g / 22 2.- 4.5 g of the liquid B. Thus, at the time T 1the vessel contains 95.5 g of the liquid B and no liquid A). The temperature t 2 is equal to the boiling temperature of the liquid B: t 2= 100°C. Harking scheme 1. physical condition for boiling 2. boiling temperature of the liquid A (numerical value) 3. boiling temperature of the liquid B (numerical value) 4. analysis of the phenomena at the temperature t 1 5. numerical value of t 1 6. numerical value of the mass ratio of the saturated vapours in the bubble 7. masses of the liquids at the time Ti 8. determination of the temperature t 2 REMARK: As the sum of the logarithms does not equal to the logarithm of the sum, the formula given in the text of the problem should not be applied to the mixture of the saturated vapours in the bubbles formed on the surface separating the liquids. However, the numerical data have been chosen in such a way that even that incorrect solution of the problem gives the correct value of the temperature t i (within required accuracy). Our purpose was to allow the
1 1 1 3
1 1 1 1
10
pupils to solve the part B of the problem even if they determined the temperature t1in a wrong way. Of course, one cannot receive any points for an incorrect determination of the temperature t even if its numerical value is correct. 1 Remarks
and the typical mistakes in the
pupils'
solutions
Nobody has received the maximum possible number of points for this problem, although several solutions came close. Only two participants tried to analyze proportion of pressures of the vapours during the upward movement of the bubble trough the liquid B. Part of students confused Celsius degrees with kelvins. Many participants did not take into account the boiling on the surface separating the liquids A and B, although this effect was the essence of the problem. Part of students, who did notice this effect, assumed a priori that the liquid with lower boiling temperature "must" be the first to evaporate. In general, this need not be true: if y were, for example, 1/8 instead 8, then liquid A would remain in the vessel, not liquid B. As regards the boiling temperatures, practically nobody had any essential difficulties.
11
Problem 2 Three non-collinear points 101, P2 and P3with known masses mi, m2and m3, interact with one another through their mutual gravitational forces only; they are isolated in free space and do not interact with any other bodies. Let a denote the axis going through the centre-of-mass of the three masses, and perpendicular to the triangle P1P2P3. What conditions should the angular velocity w of the system (about the axis a) and the distances:
P1P2 = a12
P2P3 = a23
P1P3 = a13
fulfill to allow the shape and size of the triangle P1P2P3 to remain unchanged during the motion of the system, i.e. under what conditions does the system rotate around the axis a as a rigid body? Solution
As the system is isolated, its total energy, i.e. the sum of the kinetic and potential energies, is conserved. The total potential energy of the points Pl, P2and P3with the masses ml, m and m in the inertial system (i.e. when there 2 3 are no inertial forces) is equal to the sum of the gravitational potential energies of all the pairs of points (P1'P ), (P2,P3) and (P1 P3). It depends only on the 2 distances a12, a and a which are constant in time. Thus, 23 13 the total potential energy of the system is constant. As a consequence the kinetic energy of the system is constant too. The moment of inertia of the system with respect to the axis a depends only on the distances from the points Pl, P2, do not P to the axis a which, for fixed a 23 and a13, 3 12, a depend on time. This means that the moment of inertia 1 is constant. Therefore, the angular velocity of the system must ,
12 also be constant:
w = const.
(1)
This is the first condition we had to find. The other conditions will be determined by using three methods described below. However, prior to performing calculations, it is desirable to specify a convenient coordinates system in which the calculations are expected to be simple. with the Let the positions of the points Pl, P2 4 masses mi, m2and m3be given by the vectors ri, r2 and r3. For simplicity we assume that the origin of the coordinate system is localized at the mass center of the points 131, P2 and P3 and that all the vectors r r 2 and r3 are in the same coordinate plane, e.g. in the plane (x,y). Then the axis a is the axis z. In this coordinate system, according to the definition of the mass center, we have:
m lrl + m 2r2 + m3r3= 0.
First
(2)
method
Consider the point P1with the mass m . The points P 1 2 and P act on it with the forces: 3 F21
11112 = G
(r2
r1 , 1 ' )
(3)
a12
m"1
3 (r3 - r1) 1'31 = Gm133 a3
,
(4)
13
where G denotes the gravitational constant. In the inertial frame the sum of these forces is the
13 centripetal force
Pr1 — ml.w r1' which causes the movement of the point P1 along a circle with the angular velocity w. (The moment of this force with respect to the axis a is equal to zero.) Thus, we have:
(5)
F21 1- P31 = Pr1.
In the non-inertial frame, rotating around the axis a with the angular velocity w, the sum of the forces (3), (4) and the centrifugal force 2-4 r1 = miw r1 should be equal to zero:
P21 +
P31
(6)
+r1 = 0.
(The moment of this sum with respect to any axis equals to zero.) The conditions (5) and (6) are equivalent. They give the same vector equality:
G
m13 m1m2 (r G 3 2 -r1) + 3 (r3- r1) + mico ri =0, a12 a13 M
1
Gm3
4 Gm M1 G I = 0. (7) m3; 3 + m1; 1 [ ( ' )2 - 32 - 3 G -7-m2r2 4a12 a13 a12 a13
From the formula (2), we get:
14
m 2r2
m1r1
(8)
m3r3'
Using this relation, we write the formula (7) in the following form: G m1 (3 a12
- m3r3) + G
mi
m3r3 +
a13
+
Gm 2
[ m1r1
2 -
7-
-
a12
Gm3 Sa13
= 0
-
i.e. [ r
1m1
2 6)
Gm2Gm3Gml (7- - 3 13 a12 12 .
7,
+ it3( 4- - 4- I Gralm3= 0. a13 a12
The vectors r1 and r3 are non-collinear. Therefore, the coefficients in the last formula must be equal to zero: [ 1 _ 1 3 a13 a12 2 GI%
i
m'1
[
w
3 a12
Gmim3 = 0,
Gm3 Gm 1 3 - 3
a13
)
= O.
a12
The first equality leads to: 1 3 a13
1 a ' a12 3
and hence,
a 13 = a12.
Let a
13
= a
12
= a. Then the second equality gives:
15 w 2a3 = GM,
(9)
where
= m + m + m 1 2 3
(10)
is the total mass of the system. In the same way, for the points P2and P3, one gets the relations: a) the point P2: w 2a3 = GM.
a 23 = a12,
b) the point P3: w 2a3 = GM.
a 13 = a23,
Summarizing, the system can rotate as a rigid body if all the distances between the masses are equal:
a12
a13
a23 a,
the angular velocity w is constant and the relation (9) holds. Second
method
At the beginning we find the moment of inertia / of the system with respect to the axis a. Using the relation (2), we can write:
16 0 = (mir. + m2-r 2
4 ,2 m3r3' =
.3 .4 242 242 m2r2 + m3r3+ 2m1m2rir2+ 2m2m3r2r3+ 2m3m1r3ri.
242 = Miri Of course:
2 42 r.1 = r-
(i = 1, 2, 3).
The quantities 21-.2-jcan be determined from the following evident relation: 2 a. j =
2
1
4 2
1=(1.--1"
=
4 4 42 42 ri + rj- 2riri,
(i,j = 1, 2, 3). We get: 4 4 2 42 42 ij 2r. 1r. 3 = r1+ r. - a .
this the help of With transformations, we obtain:
relation,
simple
after
4 2 0 = (m r + m r + m r ) = 2 2 1 1 3 3
= (ml
2 m2 1- m3)(m1r1
2 m2r2
2 m3r3)
m -
im
ja ij' i<j
The moment of inertia I of the system with respect to the axis a, according to the definition of this quantity, is
17 equal to 2 2 "2r 2
=
2 ift3r3'
The last two formulae lead to the following expression: m1 3 2 13 m
I =
i<j where M = m + m + m is the total mass of the system. l 2 3 In the non-inertial frame, rotating around the axis a with the angular velocity w, the total potential energy Vtot is the sum of the gravitational potential energies M.M.
V
.
13
1 = - u ----; a.. 13
i,j = 1, 2, 3; i<j
of all the masses and the potential energies 1 2 mir 2 i V i = -7
i = 1, 2, 3
;
of the masses in the field of the centrifugal force: 3 V
tot
-
-
G
i<j
a
13
('t. 3 i<j
-
2 m1r 1 1=1
1 w2 / = 2
13
.m m 13_ 1 w2 1 )1 m,m_a2 =
=
fz-J =
1 2 2 w
1 3
-
a
1 j 17'3
2
M
1 3
i<j
ij w 2 a.2 . + 2M 13
a. . 13
18 i.e.
V
tot
2 wn, a. 2. + m.m . [7. 3 z n 13
= i<j
G a.. ]'
13
A mechanical system is in equilibrium if its total potential energy has an extremum. In our case the total potential energyVtotis a sum of three terms. Each of them is proportional to: 2
f(a)
w =
2" G a
.
The extrema of this function can be found by taking its derivative with respect to a and requiring this derivative to be zero. We get: 2
1-w4- a - 7=
0.
a
It leads to: 0)2a3 = GM
or
w23 a = G (m1
2" 3). "
We see that all the terms in Vtot have extrema at the same values of aij = a. (In addition, the values of a and w should obey the relation written above.) It is easy to show that it is a maximum. Thus, the quantity V tot has a maximum at a. . = a. 13 This means that our three masses can remain in fixed distances only if these distances are equal to each other:
a and if the relation
12
= a
13
= a
23
= a
19 w 2a3 = GM, where M is the total mass of the system, holds. We have obtained the conditions (9) and (11) again. Third
method
Let us consider again the point P with the mass m and 1 1 the forces f and '31 given by the formulae (3) and (4). It 21 follows from the text of the problem that the total moment (with respect to any fixed point or with respect to the mass center) of the forces acting on the point P1 must be equal to zero. Thus, we have:
F21
x
1
+
f31
x
1
= 0,
where the symbol x denotes the vector product. Therefore, m1m3 m1 2 -4 1) x ;1 = P. G --7G --7 (r3 - ; (r2 - r1) x ;1 + a 13 a12 -
But
r1 x r1 = 0, thus, 1712 3
a12
4
r2
m3 x r1 +
3
r1 -
r x 3
0.
a13
Using the formula (8), the last relation can be written as follows:
20
-ai2
-
( -
1 1
1 -o -7 3- r3 x r a13
) x r1 + 'TL 3 3
3 4 3 - -7- r3 x ; 1 + -7- r3 x ri = 0, a12 a13
1 -7- - -7- ) r3 x ri [1 a13 a12 4
The vectors r
1
o.
4
and r3 are non-collinear. Therefore, 4
4
r1 x r3
0
and 1 3 a13
1 3 = 0, a12
hence,
a12
= a13.
a12
. a23
Similarly, one gets:
We have rederived the condition (11). Taking into account that all the distances a _ . have the 1] same value a, from the equation (7) concerning the point P 1' we obtain:
6
ml _ m13 2 4 (r r ) + 6 (r - r1) + m1 w r1 = 0, 2 1 a3 a3
21
G
3 2 1 (m r + m3r3) G 7 m iri G 7 m 1r1 +mlw r1 = 0, 2 2 a a a3
_
6
1
a
G m 2
a
G
3 1 m m1r1
r1 = °'
3 = w2.
GM a
This is the condition (9). The same condition is got in result of similar calculations for the points P2and P3. The method described here does not essentially differ from the first method. In fact they are slight modifications of each other. However, it is interesting to notice how application of proper mathematical language, e.g. the vector product, simplifies the calculations. Marking
scheme
1. the proof that w = const 2. the conditions at the equilibrium (conditions for the forces and their moments or extremum of the total potential energy) 3. the proof of the relation a.1 . = a w2 a3 = GM 4. the proof of the relation
1 (2)
3 (2) 4 2
(The points in brackets were suggested by the organizers of the Olympiad; the solutions were graded according to the point distribution without brackets, accepted by the International Board) Remarks solutions
and
the typical. mistakes in the
pupils'
22 No type of error was observed as predominant in the pupils' solutions. Practically all the mistakes can be put down to the students' scant experience in calculations and general lack of skill. Several students misunderstood the text of the problem and attempted to prove that the three masses should be equal. Of course, this was impossible. Moreover, it was pointless, since the masses were given. Almost all the participants tried to solve the problem analyzing equilibrium of forces and/or their moments. Only one student tried to solve the problem by looking for a minimum of the total potential energy (unfortunately, his solution was not entirely correct). Several participants solved the problem using convenient reference system: one mass in the origin and one mass on the x—axis. One of them received a special prize.
23
Problem 3 This problem concerns the investigation of transforming the electron microscope with magnetic guiding of the electron beam (which has been accelerated through a potential difference of U = 511 kV) into a proton microscope (in which the proton beam has been accelerated through a potential difference of -U). For this purpose, solve the following two problems: A) An electron after leaving a device that accelerated it with the potential difference U falls into a region with an inhomogeneous magnetic field generated with a system of stationary coils L1, L2, , Ln. The known currents in the coils are i 1, i 2; , i n, respectively. i ' in the What should the currents i 1' ' ' 2' ' n coils L1, L2, ... Ln be, in order to guide a proton (initially accelerated with the potential difference -U) along the same trajectory (and direction) as that of the electron?
A
HINT: The problem can be solved by finding a condition under which the equation describing the trajectory is the same in both cases. It may be helpful to use the relation: d
P dt p =
1 d
7 dt
_ 1 d P
-
7 dt
2 P
B) How many times would the resolving power of the above microscope increase or decrease if the electron beam were replaced with the proton beam? Assume that the resolving power of the microscope (i.e. the smallest distance between two point objects whose circular images can be just separated) depends only on the wave properties of the particles.
24
Assume that the velocities of the electrons and protons before their acceleration are zero, and that there is no interaction between the own magnetic moment of either the electrons or protons and the magnetic field. Assume also that the electromagnetic radiation emitted by the moving particles can be neglected. NOTE: Very often physicists use 1 electron-volt (1 eV), and its derivatives such as 1 keV or 1 MeV, as the unit of energy. 1 electron-volt is the energy gained by the electron that passed the potential difference equal to 1 V. Perform the calculations assuming the following data: 2 rest energy of electron E e = mec = 511 keV, E = m c 2 = 938 MeV. rest energy of proton SoLution
PartA
At the beginning one should notice that the kinetic energy of the electron accelerated with the potential difference U = 511 kV equals to its rest energy Eo. Therefore, at least in the case of the electron, the laws of the classical physics cannot be applied. It is necessary to use relativistic laws. The relativistic equation of motion of a particle with the charge e in the magnetic field t has the following form: d
71T P where and
p = m
= FL,
yv denotes the momentum of the particle (vector)
o
25
FL= e v x t is the Lorentz force (its value is evB and its direction is determined with the right hand rule). In the above formulae m is the (rest) mass of the particle and v denotes the o velocity of the particle. The quantity r is given by the formula:
_
1
i/1 - 1,2/c2
The Lorentz force FL is perpendicular to the velocity 4 4 the particle and to its momentum p = m rv. Hence,
of
o
FL v =
; = 0
.
Multiplying the equation of motion by p and making use of the hint given in the text of the problem, we get: 1 d
7 RT P
2
, =
It means that the value of the particle momentum (and the value of the velocity) is constant during the motion:
p = mory = const,
(1.) = const).
The same result can be obtained without any formulae in the following way: The Lorentz force FL is perpendicular to the velocity v (and to the momentum p as p = morv) and, as a consequence, to the trajectory of the particle. Therefore, there is no force which could change the component of the momentum tangent to the trajectory. Thus, this component, whose value
26 is equal to the length of p, should be constant: p = const. (The same refers to the component of the velocity tangent to the trajectory as p = morv). Let s denotes the path passed by the particle along the trajectory. From the definition of the velocity, we have: ds dt _ U.
Using this formula, we can rewrite the equation of motion as follows: d ds P
ds d dt ds P
d dt P
'L.
FL
d
ds P v •
Dividing this equation by p and making use of the fact that p = const, we obtain:
f
d 2 = L ds p vp'
and hence d
ds
FL
2
vp'
where t = p/p = v/v is the versor tangent to the trajectory. The above equation is exactly the same for both electrons and protons if and only if the vector quantity: FL vp
is the same in both cases. Denoting corresponding quantities for protons with the
27 same symbols as for the electrons, but with primes, one gets that the condition, under which both electrons and protons can move along the same trajectory, is equivalent to the equality:
P
L u p
FL v'1,' •
However, the Lorentz force is proportional to the value of the velocity of the particle, and the directions of any two vectors of the following three: t (or FL, B determine the direction of the third of them (right hand rule). Therefore, the above condition can be written in the following form: ;),
e B _ e'B'
Hence,
This means that at any point the direction of the field g should be conserved, its orientation should be changed into the opposite one, and the value of the field should be multiplied by the same factor p'/p. The magnetic field g is a vector sum of the magnetic fields of the coils that are arbitrarily distributed in the space. Therefore, each of this fields should be scaled with the same factor -p'/p. However, the magnetic field of any coil is proportional to the current flowing in it. This means that the required scaling of the fields can only be achieved by the scaling of all the currents with the same factor p'/p: -
n
_ EL p
n'
28 Now we shall determine the ratio p'/p. The kinetic energies of the particles in both cases are the same; they are equal to Ek =eIUI = 511 keV. The general relativistic relation between the total energy E of the particle with the rest energy E0and its momentum p has the following form: £ 2 = E 2 + p2c 2,
where c denotes the velocity of light. The total energy of considered particles is equal to the sum of their rest and kinetic energies:
E = E
o
+ E
k'
Using these formulae and knowing that in our case Ek
=
= eIUI = E , we determine the momenta of the electrons (p) e and the protons (p'). We get:
a) electrons:
(E
e
+ E ) 2 = E 2 + p 2c 2 , e
E
P =
Fe
b) protons: (E + E ) 2 = £2 + p' 2c 2, P e
2 P' = ce 4 Hence,
E P
1
2 EP
)
29
2 /[ _2 + 1 0) Ee
1
P
2
)
35. ,
and
- 35.0 i . n
n
It is worthwhile to notice that our protons are 'almost classical', because their kinetic energy Ek (= Ee) is small compared to the proton rest energy E . Thus, one can expect that the momentum of the proton can be determined, with a good accuracy, from the classical considerations. We have: ,2
Ee = E k
P
1 p' = c—
_ P
,22
_ P
2% c 2
,22 '
2Ep
E eE p.
the proton On the other hand, the momentum of determined from the relativistic formulae can be written in 1. We get: a simpler form since E p/E e E P = c
i//( E E
2 p
1
e
Ee i r
-
[EP
= -c
)
Ep
2 E
+ 1
e
E 1// E e p 2 E c e
1
- c
E eE p ,
In accordance with our expectations, we have obtained the same result as before. PartB
The resolving power of the microscope (in the meaning
30 mentioned in the text of the problem) is proportional to the wavelength, in our case to the length of the de Broglie wave:
= — p'
where h denotes the Planck constant and p is the momentum of the particle. We see, that k is inversely proportional to the momentum of the particle. Therefore, after replacing the electron beam with the proton beam the resolving power will be changed by the factor p/p' z 1/35. It means that our proton microscope would allow observation of the objects about 35 times smaller than the electron microscope. Marking scheme
1. the relativistic equation of motion 2. independence of p and v of the time 3. identity of et/p in both cases 4. scaling of the fields and the currents with the same factor 5. determination of the momenta (relativistically) 6. the ratio of the momenta (numerically) 7. proportionality of the resolving power to X 8. inverse proportionality of X. to p 9. scaling of the resolving power Remarks
and the typicaL
mistakes
to the
1 1 2 1 1 1 1 1
1 pupils'
Solutions
Some of the participants tried to solve the problem by using laws of classical mechanics only. Of course, this approach was entirely wrong. Some students tried to find the required condition by equating "accelerations" of particles in both cases. They understood the "acceleration" of the
31 particle as a ratio of the force acting on the particle to the "relativistic" mass of the particle. This approach is incorrect. First, in relativistic physics the relationship between force and acceleration is more complicated. It deals with not one "relativistic" mass, but with two "relativistic" masses: transverse and longitudinal. Secondly, identity of trajectories need not denote equality of accelerations. The actual condition, i.e. the identity of eB/p in both cases, can be obtained from the following two requirements: o . 1 in any given point of the trajectory the curvature should be the same in both cases; 2° 2 in the vicinity of any given point the plane containing a small arc of the trajectory should be oriented in the space in both cases in the same way. Most of the students followed the approach described just above. Unfortunately, many forgot about the second requirement (they neglected the vector character of the quantity eB/p).
32 Experimental Problem The following equipment is provided:
10 mm 1. Two piezoelectric discs of thickness 10 mm with evaporated electrodes (see figure 4.1) fixed in holders on the jaws of the calipers.
electrods Fig. 4.1.
2. Calibrated sine wave oscillator with a photograph of the control panel, explaining the functions of the switches and regulators. 3. A double channel oscilloscope with a photograph of the control panel, explaining the functions of theswitches and regulators. 4. Two closed plastic bags containing liquids. 5. A beaker with glycerin (for wetting the discs surfaces to allow better mechanical coupling). 6. Cables and a three way connector. 7. Stand for supporting the bags with the liquids. 8. Support for calipers.
A piezoelectric material changes its linear dimensions under the influence of an electric field and vice-versa, the distortion of a piezoelectric induces an electric field. Therefore it is possible to excite the mechanical vibrations in a piezoelectric by applying an alternating electric field, and also to induce an alternating electric field by mechanical vibrations. A. Knowing that the velocity of longitudinal ultrasonic waves in the disc material is about 4*103m/s, estimate
33
roughly the resonant frequency of the mechanical vibrations parallel to the disc axis. Assume that the disc holders do not restrict the vibrations. (Note that other types of eigen vibrations with lower and higher frequencies occur in the discs). estimation, determine Using your experimentally the frequency for which the piezoelectric discs work best as a transmitter - receiver set for ultrasound in the liquid. Wetting surfaces of the discs before putting them against the bags improves the ultrasound penetration. B. Determine the velocity of ultrasound for both liquids without opening the bags and estimate the error. C. Determine the ratio of the ultrasound velocities for both liquids and its error.
Complete carefully the synopsis sheet. Your report should, apart from synopsis sheet, contain the descriptions of:method of resonant frequency estimation,methods of measurements, methods of estimating errors of the measured quantities and of the final results. Remember to define all the used quantities and to explain the symbols.
Synopsis sheet A. Formula for estimating the resonant frequency
result (units)
34
Measured best transmitter frequency (units)
B. Definition of measured quantity
symbol
results (units)
Final formula for ultrasound velocity in liquid:
Velocity of ultrasound (units)
Liquid A
Liquid B
error
error
error
35
C. Ratio of velocities
error of the ratio
Solution of the Experimental Problem A.1. As the holders do not restrict the discs vibrations one expects antinodes on the flat surfaces of the discs and a node in the middle of the discs height (Figure 4.2). One resonant frequency is expected for 1= A/2 = v/2f where v is the velocity of longitudinal ultrasound wave (4.103m/s), X is the wavelength, f the frequency and 1 is the thickness of the disc. So f = v/21 = 4.103/2.10-2= =2.105
// //////
Hz. This is only a rough estimation as the diameter of the disks is not much bigger then its thickness and the influence of piezoelectric effect on ultrasound velocity is neglecte.
=1 Xx 2
Fig. 4.2 The experimental set-up is shown in fig. 4.3.
Fig. 4.3
36 The oscillator is connected to one of the discs which works as a transmitter and to one channel of the oscilloscope. The second disc is connected to the second channel of the oscilloscope forming the receiver. Both discs are placed against one of the bags containing liquid (fig. 4.4) and the frequency
7177777/7/7777/7/777/7/%7 Fig. 4.4 is changed slowly in the 100-1000 kHz range. For resonant frequency f », 220 KHz the amplitude of the received signal shows a sharp peak. One also finds peaks of received amplitude at f of about 110 kHz and 670 KHz. There are two ways of determining the optimum frequency at the peak. One is the direct value from the oscillator dial giving the accuracy of 10 kHz (1/2 division), the second is the value taken from the oscilloscope using the time base calibration (accuracy 5 kHz if the length of ten periods is measured with 1/2 division accuracy.) B. Using the same set-up as in figs. 4.3 - 4.4 one measures the dependence of the phase shift between the signals in
37
channels one and two of the oscilloscope (fig.4.5) on the distance d between the
f
Y
X
Fig. 4. 5 Fig. 4.6 piezoelectric discs. The transmitted ultrasound waves pass through the liquid and generate the electric signal in the receiver, phase shifted by Aly) = 2irdf/v. For d=v/f=nA where n is an integer the phase difference becomes 2irn which is easily observable on the oscilloscope screen. To obtain a steady picture on the oscilloscope it is essential to trigger the time base from one of the signals preferably the one from the receiver. The other way of determining the zero phase shift points would be to utilize the x-y mode of oscilloscope (without the time base).The picture obtained is shown in fig. 4.6 (the transmitter signal is connected to the x input, and the receiver one to y input amplifier). At AT=0±2y and r t2r the eclipse becomes a straight line. Not all the available oscilloscopes have this mode of operation. Fig. 4.6 shows the dependence of d on n for liquid A (water) and Fig. 4.7 for liquid B (glycerin). One can draw a straight line through the points finding
38
and AA = 0.66 .1 0.05 cm AB = 0.86 t. 0.05 cm which gives VB = (1.96 1.0.10).103 m/s VA = (1.50 .1 0.10).103 m/s and the ratio n = 1.31 1.0.15 Much better accuracy of measurements can be obtained if after the introductory phase shift observation one concentrates on the difference of two zero phase shift points and measures the corresponding distances di and d2 several times slightly changing the shape of the bag containing the liquid before each measurement. This procedure averages out the influence of ultrasound waves scattered from walls of the bags which affects the observed phase shift.
L[cm] 10.0
9.0
8.0
7.0
5.5
0
1
2
3
4
5 n
Fig. 4.7 The sample results for dl-d2=4A are given in the table 1.
39 Table 1 Liquid A (transparent)
Liquid B (yellow)
#
di
d2
di
d2
1
9. 10
6. 29
8. 79
5. 24
2
9. 12
6. 28
8.75
5. 19
3
9. 09
6. 26
8. 81
5. 19
4
9. 09
6. 31
8. 83
5. 24
5
9. 10
6. 24
8. 81
5. 21
6
9. 07
6. 32
8. 75
5. 18
7
9. 09
6. 28
8. 80
5. 22
8
9. 07
6. 28
8. 76
5. 22
9
9. 11
6. 25
8. 85
5. 22
10
9. 07
6. 29
8. 80
5. 19
11
9. 08
6. 30
8. 74
5. 16
12
9. 09
6. 32
8. 78
5. 19
13
9.09
6. 28
8. 76
5. 26
L[cm] 4 10.0 Fluid B ( yellow)
9.0
GLICERIN 8.0
7.0
5,5
Ow.
0
1
2 Fig. 4. 8
3
4
40
From these results one obtains the values Fluid A = 6.285 1 0.007 = 9.090 10.095 Fluid B = 8.786 1 0.009 = 5.208 1 0.008 Putting in f = (215 f 3)•103Hz one obtains vA = (1508 ± 24) m/s vB = (1923 1 30) m/s and the ratio 1.276 t 0.006 It should be pointed out that the error of frequency which is the prevailing factor in the error of velocity doesn't affect the error of the ratio. Marking of the experimental problem: Frequency estimation Ai. Formula A2. Result(with units) A3. Method of measurement of the best transmitting frequency A4. Result (if within 10% of standard value-1p) (if within 5% of standard value-2p) A5. Correct error estimation of the above B1. Clear explanation of the method of the measurement of ultrasound velocity B2. Series of distances d measured (if less then 5 results for ci1 d2 for both liquids are obtained 1/3 of a point is subtracted from 3 for each missing measurement) -
1p 1p ip 2p
1p
2p 3p
41
B3. Correct result for VI
2p
(if within 5% of standard value-2p) (if within 15% of standard value-1p) B4. Correct error estimation of the above B5. Correct result for VB
ip 2p
(if within 5% of standard value-2p) (if within 15% of standard value-1p) B6. Correct error estimation of the above C1. Correct result for VA/VB
ip 2p
(if within 3% of standard value-2p) (if within 10% of standard value-1p) C2. Correct error estimation of the above
ip
It should
be pointed out that all the experimental set-ups were carefully checked before competition and the best transmission frequencies were measured ,using both the oscilloscope and oscillator (and then as a check of calibration ratemeter), for all of them separately. These values were then compared with the results obtained by competitors. In case of discrepancies due for example to the fact that equipment was replaced during the competition control measurements were performed. The set-ups were available until the end of marking and discussion with leaders. The results of experimental problem were very good. More then a half of the competitors obtained more than 15 points. Many students however had troubles with frequency estimation. Some for example were assuming that nodes are formed on the flat surfaces of the disks (this was accidentally leading to the proper formula but was not accepted as the model was wrong). In part B some students were trying to measure the distances between nodes and
42
antinodes of the ultrasonic standing wave formed in the liquid. This was not giving proper results as the nodes and antinodes pattern was changing in an unpredictable way once the distance between disks and at the same time shape of the bag is changed.
43
Histograms
of marks
Cnumber of solutions vs. number of points obtained, 68
//
Problem. 1
59•
0
i.
48
//
0
38 0 0
20 -
18•
9 8
0
0 0
•
44
Problem 3 /A
////
/7/
//
C 0
Points 11
/
r P/ 0 /./
10 9 8
Theoretical part
//, Vi
C three probterns,
C C
1
co 4o
P
•
3
6
g'
4
5 4
■
/
/ /
'
I
/
A
3
/
/ 7
'
;/,
2
H '
7/
/ 00
/ 1
;/ /
,,/,`/
6
/ /, /
.,
,
,/
I
Points
(4 '74
r ,,, ;r,, .:„,71,./.../.„,..., Ay, 2: 1,; j//1 fA /• //1/,//,.,A.,,
l.,11!" ,0 '% %/%.1'. ' A 14
6
/ 4f / 1 77 /,/ ',//; ./ ;, ,i
//,
12
/1
,/,, Iir
1/'4,4/ .
1 -
;/
.
/// / $
r 'A /ffi V. / VAMP /A 0 ,IY/ 1/1r/e'-rri `/A r,'Ar, Yr/ ZA 8 2 4 18
,;
16
18
/-A 1'i:A IZ,1/,',4m
28
Z..'
24
ri.A v;`, ', 25
28
313
45 18 Experimental part Cone problem) 16 14 12 10 8 /
6-
//
/
4
/( 14
16
18
Points 7
Total scores
1 M 1 '
0
y
0
41
.
I
0 V I
Y1..
A
-r R 1 1; rllV 16N In ^ ' ' 1, A 0 H KA 1,m, (//0 /00 ,/ imK, 116 K61 AV 6
./.
2
, 1
6
'
I
-,,,,,,,
"4//
0 2 4 6 8 0121416
/'
r%
r
i
161
%
/
i
8212224 .'628303234 36 38 40 42 44 46 48 50 Points
46
MINUTES of the sessions of the International Board held during the XX International Physics Olympiad in Warsaw in July 16 — 24, 1989
1. 1. The Organizing Committee invited, following the suggestion of the Lithuanian Physical Society, the team from the Lithuanian SSR to the XX IPhO as an unofficial participant. The International Board agreed to accept the unofficial participation of the Lithuanian team on the following conditions: a) the papers of the Lithuanian participants were to be marked by the marking teams; b) their results will not be introduced into the official classification list; c) the Lithuanian participants will receive special kinds of diplomas and/or certificates different than other participants. 1. 2. The leaders of the team from the Lithuanian SSR presented to the International Board the letter by Professor H. Zabulis, Public Education Minister of the Lithuanian SSR, asking the International Board to consider in the future the full participation of the Lithuanian SSR in the IPhO. The International Board has decided to enclose the letter into the Minutes and to postpone the decision on this matter to the next Olympiad (in Holland) so that the delegation leaders can consult the problem with appropriate national authorities
47 (voting: for - 50, against - 1, abstained - 2). 2. 1. The International Board has introduced by the qualified majority of voices the following changes into the Statutes: a) # 2: the words ... participants of the previous competitions ' were changed into the words' participants of any of the Last three competitions b) # 4: the following sentence was added after the first sentence:
Also students who finished their school
examination in the year of
the
competition can
members of a team as Long as they do not
be
start the
university studies'. c) # 16: the following paragraph was added at the end of
this chapter:
No changes may be made to these
Statutes or Syllabus unless each delegation obtained
written text of the proposal at Least three months in advance. '
2. 2. The International Board did not accept suggested changes on: publication of the proceedings, languages in which the texts of probles and their solutions have to be prepared, non-participation of the golden medalists in the following Olympiads, the establishing of the Appeal Board. 2. 3. The International Board has accepted with the qualified majority the idea of giving formal recognition to the secondary school teachers of the International Physics Olympiad students. It was agreed that this formal recognition will be given during the XXI IPhO, however, a final formulation of the appropriate point in the Statutes will be discussed and confirmed at the
48 International Board meeting in 1990. 2. 4. The representatives of Finland presented to International Board the following change into Statutes concerning the participation fee (to # 6):
the the
.* To cut the costs of hosting the IPhO as well as to lower the financial threshold of raising money in the host country there is a Cron-compulsory) fee for the IPhO. The
minimal
International Board.
fee should be decided by the The
fee
should be paid in
advance to the account specified by the host country. A country which has hosted the Olympiad is freed from paying the fee during five subsequent IPhO following its own Olympiad.'
for consideration. 3. The Conclusions and Recommendations of the Unesco Consultation on Future Development of Science Olympiads (Enschede, Holland, 22nd - 24th March 1989) were presented and discussed. One recommendation concerning the teachers was accepted (see 2.3). The recommendation concerning the golden medalist was rejected and the other two recommendations were not voted upon as they concern the practice established in the IPhO a long time ago. The International Board has decided unanimously that Dr. Waldemar Gorzkowski (Poland). the Secretary of the IPhO, should represent the International Physics Olympiad as a member of the Committee of the International Science and Mathematics Olympiads which is going to be established by Unesco. 4. The International Board was pleased to accept the creation of the yearly award founded by the European Physical Society for the student who obtains best balanced theoretical and experimental results.
49
As Dr. Lars Silverberg (Sweden) resigned his position of the permanent representative of the International Physics Olympiad to the European Physical Society, due to retirement, the International Board has unanimously elected to this post Dr. Lars Gislen (Sweden). 5. As the term of office of Vice-Secretary of the International Physics Olympiad was finished this year the International Board has unanimously reelected Dr. Andrzej Kotlicki (Poland) to this post for the period 23rd July, 1989 - 22rd July,1994. the 6. Dr Guy Bagnall agreed to collect suggestions concerning the rules of marking the olympic problems which will be discussed next year. The dead line for sending him proposals, comments and suggestions concerning this matter was established to be December 31, 1989. 7. The preliminary column version of the Syllabus was accepted by qualified majority as obligatory for the next year. The final version is to be prepared and discussed in 1990. The dead line for sending to the Secretariat of the IPhO the proposals and comments on this matter was established to be December 31, 1989. The new version is to be sent to all the delegation leaders by March 31, 1990. 8. Dr. S. Krotov (Soviet Union) filed the official protest stating that not all the content of the discussions was translated into the Russian at the beginning of the sessions of the International Board. 9. The results of marking the papers by the organizers were accepted. The following limits for awarding the medals
50 were established according to the Statutes: 41 35 29 22
points points points points
-
Gold Medal Silver Medal Bronze Medal Honourable Mentions
According to these limits 10 Golden Medals, 26 Silver Medals, 30 Bronze Medals and 33 Honoureable Mentions were awarded. The best score (46.333 points) was achieved by Steven Gubser (USA), the second best (45.5) by Szabolcs Kesmarki (Hungary). Apart from the special prizes for the best and second best scores the following prizes were awarded: for the best solution to the problem 1: Desmond Roney Lim Chin Siong (Singapore) for the best solution to the problem 2: Gabriel Balan (Romania) for the best solution to the problem 3: Ge Ning (China) for the best solution to the experimental problem: Romke' Jonker (Holland) Colin Merryweather (Great Britain) for the best girl: Mona Berciu (Romania) for the youngest participant: Jessica Millar (USA) for the best combined score to the problem nr 1 and th experimental problem (founded by the Institute of Low Temperatures and Structural Research in Wroclaw, Poland): Colin Merryweather (Great Britain) for the best balanced experimental and theoretical solutions (found by the European Physical Society):
51 Andrej Dobot(Czechoslovakia) for the most original solution (found by Professor I. Bialynicki-Birula, the winner of the 1st Polish Physics Olympiad): Gabriel Balan (Romania) 10. Dr. Hans Jordens, the head of the Dutch team, has announced that the next International Physics Olympiad will be organized in Groningen (The Netherlands) on July 5 - 14, 1990 and cordially invited all the participating countries to attend the competition. Secretary of the International Physics Olympiads
Vice-Secretary of the Intern. Physics Olympiads
,44ee14:
6)SktiLelLth, Dr. W. Gorzkowski
Acting President of the XX Intern. Physics Olympiads
Dr. A. Kotlicki
rof. J. Blinowski
Warsaw, July 24, 1989
encl.:the letter by Professor H. Zabulis, Public Education Minister of the Lithuanian SSR
52
002754 6 0200200
LIETUVOS TSR LIAUDIES SVIETIMO MINISTERIJA MHHHCTEPCTBO HAPOAHOTO OEPA3OBAHI4g MITOBCKOla CCP 232691 Vilnius, Svietimo g. 2/7. Tel.: 622483, 616217, 612590, 612107, 612025, 612156. Teletaipas 261415 „Regula" Atsisk. saskt. Nr. 69120262, 61120105 TSRS soc. banko Vilniaus m. operacineje valdyboje
1
-
To the International Board of the International Physics Olympiads
232691, r. BRAID/HOC, yA. 1.1.1nevat4o, 2/7 Ten.: 622483, 616217, 612590, 612107, 612025, 612156. TeneTatin 261415 aPerynay. Pam c‘t. Ns 69120262, 61120105 onepannomtom ynpannetum AMA. cog. 6agka CCCP r. 1311AFAII0C Nr.
Ne Ha
We are grateful to the Organizing Committee of the XX International Physics Olympiads for the possibility to participate in this Olympiad. In Lithuania the Republical Young Physicists Olympiads have been constantly held since 1953. Ministry of Public Education of the Lithuanian SSR appeals to the International Board to allow Lithuania to become a constant member of the International Physics Olympiads. Up to 1940 the independence of Lithuania was recognized de jure by almost all existing at that time sovereign countries. Lithuania maintained top-level (embassy) diplomatic relations with USA, United Kingdom, Sweden, Germany, Finland, Italy, Poland and other sovereign countries.On the 18th of May 1989 the Supreme Soviet of the Lithuanian SSR has accepted the amendmends to the Constitution of the Lithuanian SSR and the Declaration on the sovereignty of Lithuania. Therefore Ministry of Public Education of the Lithuanian SSR considers Lithuania to have juridical grounds to become a constant member of the International Physics Olympiads.
53 In the case when International Board complies with our request, Ministry of Public Education of the Lithuanian SSR will be responsible for the adherence to the Statutes of the International Physics Olympiads by Lithuania.
Yours sicerely
prof. H.Zabulis Public Education Minister of the Lithuanian SSR
54 The UNESCO document presented at discussed during the
XX International Physics Olympiad
UNESCO CONSULTATION ON FUTURE DEVELOPMENTS OF SCIENCE OLYMPIADS
Conclusions and recommendations 1 All participants agreed that the meeting was exceedingly They were able to exchange ideas and successful. experiences in organizing national and international olympiads. The common aim of all participants, from widely varying disciplines, is the encouragement of excellence in science among youth. 1. A number of recommendations were made by the participants
to UNESCO and ICSU: 1. 1. The Director General of UNESCO be asked to encourage the governments, of UNESCO member states, to approve and support the important out of school activities associated with International Olympiads in the basic disciplines of science and mathematics.
1
The meeting was organized by UNESCO in Enschede, The Netherlands, in March 22 - 24, 1989. UNESCO invited to the meeting a number of people active in organizing the national or international science olympiads as well as those going to organize new olympiads. Also a number of observers interested in the olympic movement was present.
55 1. 2. The creation of the Committee of International Science and Mathematical Olympiads. The independence of each olympiad in governing its own operation is paramount. The function of the committee should be to provide the following: Cf..)
exchange of information among existing olympiads
Cif., advise for the development of new international competitions in science and mathematics a forum for discussing the olympiads.
activities
of
This should have strong links with ICSU (International Council of Scientific Unions) and UNESCO. It was recommended that the ICSU Secretariat give this proposal their consideration. 1. 3. UNESCO be asked to organize a meeting of representatives from the Mathematics, Physics, Chemistry, Informatics and Biology International Olympiads every 4 to 5 years. Should other international competitions, similar to the science olympiads, be organized meantime, their representatives may also be invited. 1. 4. An
international newsletter concerned with International Science and Mathematical Olympiads, be established with UNESCO's assistance. It was recommended that Professor P. T. O'Halloran be invited to edit the newsletter.
56
1. 5. That UNESCO and ICSU note the importance of regional and sub-regional International Olympiads in Science and Mathematics. The consultation of UNESCO on Science Olympiads acknowledges Professor Valeri Vavilov's offer on behalf of the Soviet Union that we consider the possibility of using their facilities and personnel to support the Committee of International Science and Mathematics Olympiads. number of recommendations were also made to the Olympiad Boards/Juries. These recommendations are not binding. They are:
2. A
2. 1. The national olympiad teams be given some pre-olympiad training. It should last, typically, from one to two weeks. Lengthy intensive training is to be discouraged. 2. 2. Formal recognition, for example in the form of a certificate, should be given to the secondary school teachers of the successful International Olympiad students. Their names should also be recorded in the proceedings of the International Olympiads. 2. 3. Each International Board/Jury should devote at least one of its sessions to the discussion of exercises and consider the modernisation of the scientific topics by including new concepts. 2. 4. Students who have obtained a gold medal in an International Olympiad should not participate again in that particular Olympiad.
57
OF
PRIZE-WINNERS THE XX INTERNATIONAL PHYSICS OLYMPIAD
Gold Medals: - USA Steven Gubser - Hungary Szabolcs KesmArki - Romania Costin-Radu Popescu - FRG Olaf Kummer - FRG Jens Lang - Great Britain Michael Rutter Desmond Rodney Lim Chin - Singapore Siong - Soviet Union 8. Nicolay Kuzma - Bulgaria 9. Asen Kumanov 10. Eric Cator - The Netherlands 1. 2. 3. 4. 5. 6. 7.
46.333 45.500 45.000 44.833 44.000 44.000 42.500 42.250 41.333 41.333
Silver Medals: 1. 2. 3. 4. 5. 6.
Cezary 8liwa Piotr Kossacki Yan Jing Tomasz Motylewski Andrei Vilfan Mao Yong 7. Udo Karthaus 8. Colin Merryweather 9. Andre Fraenzel 10. Gabriel Balan 11. Gregory Colyer 12. Qiu Dong Yu 13. Ge Ning 14. Mika Nystrom 15. Rolf Oldeman
-
Poland Poland China Poland Yugoslavia China FRG Great Britain GDR Romania Great Britain China China Sweden The Netherlands
40.500 40.333 39.833 39.833 39.500 39.333 39.000 39.000 39.000 38.500 38.333 38.333 37.666 37.500 37.333
58 16. Szilard Szabo 17. Arnot Kobylka 18. Volker Gebhardt 19. Gabor Felsd 20. Leopoldo Avelino Pando Zayas 21. Romke Jonker 22. Gregor Weihs 23. Alecsandr Korshcov 24. Miroslav Vicher 25. Konstantin Stefanov 26. Werner Torsten
-
Hungary Czechoslovakia FRG Hungary Cuba
37.000 36.833 36.500 36.333 36.000
-
The Netherlands Austria Soviet Union Czechoslovakia Bulgaria GDR
36.000 35.833 35.750 35.667 35.333 35.333
-
Australia Singapore Yugoslavia GDR Soviet Union Canada
34.666 34.583 34.333 34.333 34.167 34.000 33.500 33.333 33.333 33.333 33.333 33.000 33.000 32.833 32.500 31.833 31.500 31.333 31.333 31.166
Bronze Medals: 1. Arthur Street 2. Lim Shiang Liang 3. Zvonimir Bandit 4. Thomas Wilcke 5. Konstantin Zuev 6. Nima Arkani-Hamed 7. Richard Wilson 8. Stefan Jacobsson 9. Oyvind Tafjord 10. Andrei Dobo,g 11. Jason Jacobs 12. Lucian Ciobica 13. Romuald Janik 14. Swen Wunderlich 15. Volker Springel 16. Yavor Velchev 17. Ramin Farjad Rad 18. Zoltan Hidvegi 19. Lin Xiao Fan 20. Viorel-Cristian Negoita
- Great Britain - Sweden - Norway -- Czechoslovakia - USA - Romania - Poland - GDR - FRG - Bulgaria - Iran -- Hungary - China - Romania
59 21. Carsten Deus 22. Hannes Sakolin 23. Ramin Golestanian 24. Matthew Brecknell 25. Martiin Mulders 26. Thomas Bednar 27. Dubravko Tomasovid 28. Derrick Bass 29. Hans Olav Sundfor 30. Chris Simons
-
GDR Austria Iran Australia The Netherlands Austria Yugoslavia USA Norway Canada
31.000 30.833 30.667 30.500 29.667 29.333 29.333 29.333 29.000 29.000
-
Iran Soviet Union Romania Finland Italy Yugoslavia
-
Italy Australia Belgium USA Turkey Finland Austria Cyprus Finland Australia Belgium Czechoslovakia Sweden Great Britain Sweden Finland
28.666 28.500 28.167 28.000 28.000 27.833 27.833 27.000 27.000 26.833 26.000 25.833 25.833 25.833 25.500 25.333 25.333 25.000 24.500 24.500 24.000 24.000
Honourable Mentions: 1. Vahid Borumand Sani 2. Vladislav Makeev 3. Mona Berciu 4. Timo Tarhasaari 5. Silvano de Franceschi 6. Dalibor TuZinski 7. Federico Toschi 8. Gareth Williams 9. Christophe Colle 10. James Sarvis 11. HUseyin Altun 12. Timo Rantalainen 13. Norbert Schorghofer 14. Georgios Ioannou 15. Janne KarimAki 16. Brett Munro 17. Gregory Lielens 18. Petr Duczynski 19. Simon Ekstrom 20. Gordon Ogilvie 21. Johan Axnas 22. Pekka Heino
60 23. Eric Nodwell 24. Roger Klausen 25. Stefan Piperov 26. Kristjan Ledsson 27. Gerardo A. MUNoz 28. Otso Ovaskainen 29. Juru Uvarov 30. Dragomir Nechev 31. Leszek Mencnarowski 32. Arnold Metselaar 33. Viekoslav Mladineo
-
Canada Norway Bulgaria Iceland Columbia Finland Soviet Union Bulgaria Poland The Netherlands Yugoslavia
(The last column contains the accumulated by the contestants)
total
number
23.833 23.500 23.500 23.000 23.000 22.833 22.833 22.500 22.333 22.167 22.000 of
points
Special Prizes: for the highest score Cthe absolute winner):
Steven Gubser (USA) for the
second highest score:
Szabolcs Kesmarki (Hungary) for the best solution to the problem 1:
Desmond Roney Lim Chin Siong (Singapore) for the best solution to the problem. 2:
Gabriel Balan (Romania) for the best solution to the problem 3:
61
Ge Ning (China) for the best solution to the experimental problem:
Rourke Jonker (Holland) Colin Merryweather (Great Britain) for the highest score by a female participant:
Mona Berciu (Romania) for the youngest participant:
Jessica Millar (USA) for the best combined score to the problem f experimental problem Cfounded by the Temperatures and Structural
institute
and
the
of Low
Research, Polish Academy of
Sciences, Wroclaw, Poland):
Colin Merryweather (Great Britain) for the best balanced experimental and theoretical solutions Cfound by the European Physical Society):
Andre] DobcA (Czechoslovakia) for the most original solution (found by Professor 1. Biarynichi-Birula, the winner
of
the
Olympiad):
Gabriel Balan (Romania)
1st Polish
Physics
62
STATUTES OF THE INTERNATIONAL PHYSICS
OLYMPIADS
(Adopted in Sigtuna, Sweden, June 1984; changes: Bad lschl, Austria, June 1988; Warsaw, Poland, July 1989)
§
1
In recognition of the growing significance of physics in all fields of science and technology, and in the general education of young people. and with the aim of enhancing the development of international contacts in the field of school education in physics, an annual physics competition has been organized for secondary school students, the competition is called the 'International Physics Olympiad' and is a competition between individuals. § 2 The competition is organized by the Education Ministry or another appropriate institution of one of the participating countries on whose territory the competition is to be conducted. Hereunder, the term 'Education Ministry' is used in the above meaning. The organizing country is obliged to ensure equal participation of all the delegations, and to invite all the participants of any of the last three competitions. Additionally, it has the right to invite other countries. Within five years of its entry in the competition a country should declare its intention to be the host for a future Olympiad. This declaration should propose a timetable so that a provisional list of the order of
63 countries willing to arrange Olympiads can be compiled. A country which refuses to organize the competition may be barred from participation, even if delegation from that country has taken part in previous competitions. § 3 Education Ministries of the participating The countries, as a rule, assign the organization, preparation and execution of the competition to a physics society or another institution in the organizing country. The Education Ministry of the organizing country notifies the Education Ministries of the participating countries of the name and address of the institution assigned to the organization of the competition. § 4 Each participating country sends a team consisting of students of general or technical secondary schools, i.e. schools which cannot be considered technical colleges. Also students who finished their school examination in the year of the competition can be members of a team as long as they do not start the university studies. The age of the participants should not exceed twenty on June 30th of the year of the competition. Each team should normally have 5 members. In addition to the students, two accompanying persons are invited from each country, one of whom is designated delegation head (responsible for whole delegation), and the other - pedagogical leader (responsible for the students). The accompanying persons become members of the International Board, where they have equal rights.
64
The delegation head and pedagogical leader must be selected from specialists in physics or physics teachers, capable of solving the problems of the competition competently. Normally each of them should be able to speak one of the working languages of the International Physics Olympiads. The delegation head of each participating team should, on arrival, hand over to the organizers a list containing personal data on the contestants (surname, name, date of birth, home address, type and address of the school attended). § 5 The working languages of the International Physics Olympiad are English and Russian. Problems and solutions have also to be translated into German and French. § 6 The financial principles of the organization of the competition are as follows: the * The Ministry which sends the students to competition covers the return travel costs of the students and the accompanying persons to the place at which the competition is held. * All other costs from the moment of arrival until the moment of departure are covered by the Ministry of the organizing country. In particular, this concerns the costs for board and lodging for the students and the accompanying persons, the cost of excursions, awards for the winners, etc.
65 § 7 The competition is conducted on two days, one for the theoretical competition and one for the experimental competition. There should be at least one day of rest between these two days. The time allotted for solving the problems should normally be five hours The number of theoretical problems should be three and the number of experimental problems one or two. When solving the problems the contestants may make use of tables of logarithms, tables of physical constants, slide-rules, non-programmable pocket calculators and drawing material. These aids will be brought by the students themselves. Collections of formulae from mathematics or physics are not allowed. The theoretical problems should involve at least four' areas of physics taught at secondary school level (see Appendix). Secondary-school students should be able to solve the competition problems with standard high school mathematics and without extensive numerical calculation. § 8 The competition tasks are chosen and prepared by the host country. § 9 The marks available for each problem are defined by the organizer of the competition, but the total number of points for the theoretical problems should be 30 and for the experimental 20. The laboratory problems should consist of theoretical analysis (plan and discussion) and experimental execution.
66
winners will receive diplomas or honourable mentions in accordance with the number of points accumulated as follows: The
The mean number of points accumulated by the three best participants is considered as 100% The contestants who accumulate more than 90% of receive first prize (diploma).
points
The contestants who accumulate more then 78% up to receive second prize (diploma).
89%
The contestants who accumulate more than 65% up to receive third prize (diploma).
77%
The contestants who accumulate more than 50% up to receive an honourable mention
64%
The contestants who accumulate less than 50% of points receive certificates of participation in the competition. The mentioned marks corresponding to 90%. 78%, 65% and 50% should be calculated by rounding off to the nearest lower integers. The participant who obtains the highest score will receive a special prize and diploma. Special prizes can be awarded. § 10 The obligations of the organizer:
67 a) The organizer is obliged to ensure that the competition is conducted in accordance with the Statutes. b) The organizer should produce a set of 'Organization Rules', based on the Statutes, and send them to the participating countries in good time. These Organization Rules shall give details of the Olympiad not covered in the Statutes, and give names and addresses of the institutions and persons responsible for the Olympiad. c) The organizer establishes a precise program for the competition (schedule for the contestants and the accompanying persons, program of excursions, etc.), which is send to the participating countries in advance. d) The organizer should check immediately after the arrival of each delegation whether its contestants meet the conditions of the competitions. e) The organizer chooses (according to § 7 and the list of physics contents in the Appendix to these Statutes) the problems and ensures the translation of the chosen problems and their solutions into the languages set out in § 5. It is advisable to select problems where the solutions require a certain creative capability and a considerable level of knowledge. Everyone taking part in the preparation of the competition problems is obliged to preserve complete secrecy. f)The organizer must interpreters.
provide the
teams with
g) The organizer must supply interpreters for the working languages who are to be available at the sessions of the International Board. The interpreters should he able to cope with physical terminology.
68
h) The organizer should provide the delegation leaders with photostat copies of the solutions of the contestants in their delegation before the final classification. i)The organizer is responsible for the grading of the problem solutions. k) The organizer drafts a list of participants proposed as winners of the prizes and honourable mentions. 1) The organizer prepares the prizes (diplomas), honourable mentions and awards for the winners of the competition. § 11
The scientific part of the competition must be within the competence of the International Board, which includes the delegation heads and pedagogical leaders of all the delegations. The Board is chaired by a representative of the organizing country. He is responsible for the preparation of the competition and serves on the Board in addition to the accompanying persons of the respective teams. Decisions are passed by a majority vote. In the case of equal number of votes for and against, the chairman has the casting vote. § 12 The delegation leaders are responsible for the proper translation of the problems from the languages mentioned in § 5 to the mother tongue of the participants.
69 § 13 The International responsibilities:
Board
has
the
following
a) to direct the competition and supervise that it is conducted according to the regulations; b) to ascertain, after the arrival of the competing teams, that all their members meet the requirements of the competition in all aspects. The Board will disqualify those The contestants who do not meet the stipulated conditions. costs incurred by a disqualified contestant are covered by his country;. c) to discuss the Organizers' choice of tasks. their solutions and the suggested evaluation guidelines before each part of the competition. The Board is authorized to change or reject suggested tasks but not to propose new ones. Changes may not affect experimental equipment. There will be a final decision on the formulation of tasks and on the evaluation guidelines. The participants in the meeting of the International Board are bound to preserve secrecy concerning the tasks and to be of no assistance to any of the participants; d) to ensure correct and just classification of the prize winners; e) to establish the winners of the competition and make and a decision concerning presentation of the prizes honourable mentions. The decision of the International Board is final; f) to review the results of the competition.
70
g) to select the country which will be assigned the organization of the next competition. Observers may be present at the meetings of the International Board, but not to vote or take part in the discussion. § 14 The institution in charge of the Olympiad announces the results and presents the awards and diplomas to the winners at an official gala ceremony. It invites representatives of the organizing Ministry and scientific institutions to the closing ceremony of the competition. § 15 The long term work involved in organizing the Olympiads is coordinated by a 'Secretariat for the International Physics Olympiads'. This Secretariat consists of a Secretary and Vice—Secretary normally from the same country. They are elected by the International Board for a period of five years when the chairs become vacant. § 16 The present Statutes have bees drafted on the basis of experience gained during past international competitions. Changes in these Statutes, the insertion of new paragraphs or exclusion of old ones, can only be made by the International Board and requires qualified majority (2/3 of the votes). No changes may be made to these Statutes or Syllabus
71 unless each delegation obtained written text of the proposal at least three months in advance. § 17 Participation in an International Physics Olympiad signifies acceptance of the present Statutes by the Education Ministry of the participating country. § 18 The originals of these Statutes are written in English and Russian.
******************************** ***************************
(see
the Minutes - point 2.3)
Proposed wording of the change into the Statutes concerning the recognition of the teachers (to be introduced as a last paragraph of the Chapter # 9): Formal recognition in the form of certificate should be given to the secondary school teacher of the International Physics Olympiad students. The list of the teachers (one per each student) to be recognized should be given to the organizers by the team leaders not later than on the arrival of the team.
72 Appendix to the Statutes of the International Physics Olympiads
THE
SYLLABUS
(The column version of the 1985/86 Syllabus with slight modifications, preliminarily accepted in Warsaw, Poland,
in
JuLy 1989. To be discussed and accepted in the finaL form in Groningen, The Nether Lands, in July 1990,
General Adopted in Portorot, Yugoslavia, June 1985
a) The extensive use of the calculus (differentiation and integration) and the use of complex numbers or solving differential equations should not be required to solve the theoretical and practical problems. b) Questions may contain concepts and phenomena not contained in the Syllabus but sufficient information must be given in the questions so that candidates without previous knowledge of these topics would not be at a disadvantage. c) Sophisticated practical equipment likely to be unfamiliar to the candidates should not dominate a problem. If such devices are used then careful instructions must be given to the candidates. d) The candidates should know the system of units used in the country of origin (the original texts of the problems have to be set in the SI units). e) The candidates should be familiar with the material
73 covered by the past problems of the International Physics Olympiads.
A. Theoretical Part Adopted in Portoro, Yugoslavia, June 1985
The first column contains the main entries whiLe the second column contains comments and remarks if necessary.
1. Mechanics a) Foundation of kinematics of Vector description of the of a point mass position of the the point mass, velocity and acceleration as vectors b) Newton's laws, inertial systems
Problems may be set on changing mass
c) Closed and open systems, momentum and energy, work, power d) Conservation of energy, conservation of linear momentum, impulse e) Elastic forces, frictional forces, the law of gravitation, potential energy and work in a gravitational field
Hooke's law, coefficient of friction (F/R = const), frictional forces static and kinetic, choice of zero of potential energy
74 f) Centripetal acceleration, Kepler's laws
1
2. Mechanics of Rigid Bodies a) Statics, center of mass, torque
Couples, conditions of equilibrium of bodies
b) Motion of rigid bodies, translation, rotation, angular velocity, angular acceleration, conservation of angular momentum
Conservation of angular momentum about fixed axis only
c) External and internal forces, equation of motion a rigid body around the fixed axis, moment of inertia, kinetic energy of a rotating body
Parallel axes theorem (Steiner's theorem).
d) Accelerated reference systems, inertial forces
Knowledge of the Coriolis force formula is not required
additivity of of the moment of inertia
3. Hydromechanics No specific questions will be set on this but students would be expected to know the elementary concepts of pressure, buoyancy and the continuity law.
4. Thermodynamics and Molecular Physics a) Internal energy, work and Thermal equilibrium, heat, first and second laws quantities depending on of thermodynamics state and quantities
75 depending on process b) Model of a perfect gas, pressure and molecular kinetic energy, Avogadro's number, equation of state of a perfect gas, absolute
Also molecular approach to such simple phenomena in liquids and solids as boiling, melting etc.
temperature c) Work done by an expanding gas limited to isothermal and adiabatic processes
Proof of the equation of the adiabatic process is not required
d) The Carnot cycle, thermodynamic efficiency, reversible and irreversible processes, entropy (statistical approach), Boltzmann factor
Entropy as a path independent function, entropy changes and reversibility, quasistatic processes
5. Oscillations and waves a) Harmonic oscillations, equation of harmonic oscillation
Solution of the equation for harmonic motion, attenuation and resonance - qualitatively
b) Harmonic waves, propagation of waves, transverse and longitudinal waves, linear polarization, the classical Doppler effect, sound waves
Displacement in a progressive wave and understanding of graphical representation of the wave. measurements of velocity of sound and light, Doppler effect in one dimension only, propagation of waves in homogeneous and isotropic media, reflection and refraction, Fermats
76 'principle c) Superposition of harmonic waves, coherent waves, interference, beats, standing waves
Realization that intensity of wave is proportional to the square of its amplitude. Fourrier analysis is not required but candidates should have some understanding that complex waves can be made from addition of simple sinusoidal waves of different frequencies. Interference due to thin films and other simple systems (final formulae are not required), superposition of waves from secondary sources (diffraction)
6. Electric Charge and Electric Field a) Conservation of charge. Coulomb's law b) Electric field, potential, Gauss' law
Gauss' low confined to simple symmetric systems like sphere, cylinder, plate etc., electric dipole moment
c) Capacitors, capacitance, dielectric constant, energy density of electric field
7. Current and Magnetic Field a) Current, resistance, internal resistance of
Simple cases of circuits containing non-ohmic devices
77 source, Ohm's law, Kirchhoff's laws. work and power of direct and alternating currents, Joule's law
with known V-I characteristics
b) Magnetic field (B) of a current, current in a magnetic field, Lorentz force
Particles in a magnetic field, simple applications like cyclotron, magnetic dipole moment
c) Ampere's law
Magnetic field of simple symmetric systems like straight wire, circular loop and long solenoid
d) Law of electromagnetic induction, magnetic flux, Lenz's law, self-induction, inductance, permeability, energy density of magnetic field e) Alternating current, resistors, inductors and capacitors in AC-circuits, voltage and current (parallel and series) resonances
Simple AC-circuits, time constants, final formulae for parameters of concrete resonance circuits are not required
8. Electromagnetic waves a) Oscillatory circuit, frequency of oscillations, generation by feedback and resonance
78 b) Wave optics, diffraction from one and two slits, diffraction grating, resolving power of a grating, Bragg reflection c) Dispersion and diffraction spectra, line spectra of gases Superposition of polarized d) Electromagnetic waves as waves transverse waves, polarization by reflection, polarizers e) Resolving power of imaging systems I f)Black body, Stefan-Boltzmann's law
Planck's formula is not required
9. Quantum Physics a) Photoelectric effect, energy and impulse of the photon
Einstein's formula is required
b) De Broglie wavelength, Heisenberg's uncertainty principle
10. Relativity a) Principle of relativity, addition of velocities,
79 relativistic Doppler effect' b) Relativistic equation of motion, momentum, energy, relation between energy and mass, conservation of energy and momentum
11. Matter a) Simple applications of the Bragg equation I b) Energy levels of atoms and molecules (qualitatively), emission, absorption. spectrum of hydrogenlike atoms c) Energy levels of nuclei (qualitatively), alpha-, beta- and gamma-decays, absorption of radiation, halflife and exponential decay, components of nuclei, mass defect, nuclear reactions
B. Practical Part CAdopted in London-Harrow, United Kih6dom, July 1986)
The Theoretical Part of the Syllabus provides the basis for all the experimental problems. The experimental problems
80 in the given measurements.
experimental
contest
should
contain
Additional. requirements:
1. Candidates must be aware measurements.
that
instruments
affect
2. Knowledge of the most common experimental techniques for measuring physical quantities mentioned in Part A. 3. Knowledge of commonly used simple laboratory instruments and devices such as calipers, thermometers. simple volt-, ohm- and ammeters, potentiometers. diodes, transistors, simple optical devices and so on. 4. Ability to use, with the help of proper instruction, some sophisticated instruments and devices such as double-beam oscilloscope, counter, ratemeter, signal and function generators, analog-to-digital converter connected to a computer, amplifier, integrator, differentiator, power supply, universal (analog and digital) volt-, ohm-- and ammeters. 5. Proper identification of error sources and estimation of their influence on the final result(s). 6. Absolute and relative errors, accuracy of measuring instruments, error of a single measurement, error of a series of measurements, error of a quantity given as a function of measured quantities. 7. Transformation of a dependence to the linear form by appropriate choice of variables and fitting a straight line to experimental points. 8. Proper use of the graph paper with different scales (for
81
example polar and logarithmic papers). 9. Correct rounding off and expressing the final result(s) and error(s) with correct number of significant digits. 10.Standard knowledge of safety in laboratory work. (Nevertheless, if the experimental set-up contains any safety hazards the appropriate warnings should be included into the text of the problem.)
82
/NTERNATIONAL PHYSICS OLYMPIADS HISTORY AND PERSPECTIVES
Waldemar Gorzkowski Institute of Physics, Polish Academy of Sciences, al. Lotnikow 32/46, 02-668 Warszawa, Poland
1. SHORT HISTORY is an The International Physics Olympiad (IPhO) international physics competition for secondary school students. The first such competition was organized by Prof. Czesiaw Scislowski in Warsaw (Poland) in 1967. Since that time the International Physics Olympiads are organized, with few exceptions to be discussed later, in a different country every year. The possibility of organizing the International Physics Olympiads was speculated prior to 1967. It was clear that the International Physics Olympiads should be an annual event like the International Mathematics Olympiad, already in existence, organized for the first time in 1959. The success of the International Mathematics Olympiads, and a positive experience gained from its organization, greatly stimulated physicists involved in physics education and interested in comparison of knowledge of the best students from different countries. The hard work and dedication of Professors: R. Kostial from Czechoslovakia, R. Kunfalvi from Hungary and Cz. Scisiowski from Poland deserved particular praise. Each of them investigated various possibilities of organizing the first International Physics Olympiad in his country. It was concluded that Poland offered the best
83
conditions and the most favorable atmosphere for such an event. This, together with a great personal contribution by Prof. Czeslaw Scislowski, resulted in the first international physics competition that took place in Warsaw in 1967. One should underline here an essential difference between the International Mathematics Olympiads and the International Physics Olympiads: at the International Physics Olympiads the participants solve not only theoretical problems but also the experimental ones. For this reason the organization of the competition in physics is much more complicated and much more expensive. Several months before the first IPhO took place. invitations were sent to all the East European countries. The invitations were accepted by Bulgaria, Czechoslovakia, Hungary and Romania (five countries including Poland, the organizer of the competition). Each team consisted of three secondary school students accompanied by one supervisor. The competition was arranged along the lines of the final stage of the Polish Physics Olympiad: one day for theoretical problems and one day for carrying out an experiment. One obvious difference was the participants having to wait for the scripts to be marked. During the waiting period the organizers arranged two excursions by plane to Krakow and to Gdansk. At the first IPhO the students had to solve four theoretical and one experimental problems. The second Olympiad was organized by Prof. R. Kunfalvi in Budapest, Hungary, in 1968. Eight countries took part in that competition - German Democratic Republic, Soviet Union and Yugoslavia joined the participating countries. Again, each country was represented by three secondary school
84
students and one supervisor. Some time before the second IPhO a preliminary version of the Statutes and the Syllabus were produced. Later these documents were officially accepted by the International Board consisting of the supervisors of the teams that participated in the competition. This took place during a special meeting organized in Brno, Czechoslovakia, several months after the second IPhO. It is proper to underline that, in spite of various changes to be made later, all the basic features of the first Statutes remain valid to this day. The third IPhO was arranged by Prof. R. Kostial in Brno, Czechoslovakia, in 1969. On that occasion each team consisted of five students and two supervisors. The competition in Brno was organized according to the official Statutes accepted earlier. The next Olympiad took place in Moscow, Soviet Union, in 1970. Each country was represented by six students and two supervisors. During that Olympiad several small changes were introduced into the Statutes. Since the fifth IPhO, held in Sofia, Bulgaria, in 1971, each team consists of five pupils and two supervisors. The sixth IPhO was held in Bucharest, Romania, in 1972. It was an important event because among the participants there were present for the first time: the first out-of-European country (Cuba) and the first Western country (France). At this Olympiad the International Board decided to introduce several changes into the Statutes (however, no written proposal of the changes was produced). Unfortunately, in 1973 there was no Olympiad because no
85 country was willing to organize it, although the number of participating countries exceeded the number of the past organizers. When it seemed likely that the International Physics Olympiads would die, Poland took the initiative of reviving the international competition and organized the VII IPhO in Warsaw in 1974 (for the second time). On this occasion the Federal Republic of Germany was invited to attend the competition for the first time. This fact certainly had a symbolic significance. Before the competition. the Organizing Committee introduced into the Statutes the verbal changes discussed and accepted in Bucharest. The new version of the Statutes was sent to all the countries invited to the competition for acceptance or comments. The wording suggested by the Organizing Committee was accepted (with only one voice against). The most important changes were as follows: a) the number of theoretical problems was reduced from four to three; b) the number of working languages was reduced from four to two (English and Russian): c) there should be one day of rest between the two examination days: d) the criteria for prizes should be expressed in percentages with respect to the highest score received in given competition (formerly they had been counted with respect to the highest theoretically possible score). In 1975, 1976 and 1977 the International Physics Olympiads took place in German Democratic Republic (for the first time), Hungary (for the second time), and Czechoslovakia (for the second time), respectively.
86
In spring 1977 in Ulan-Bator, Mongolia, there was a Conference of the Ministers of Education of Socialist Countries. The Conference decided that the socialist countries would organize the International Chemistry, Mathematics and Physics Olympiads every two years. This decision was a consequence of the increasing number of participating countries and rapidly increasing organizational costs. The above decision was commonly interpreted an implicit invitation to other countries to take charge of the international scientific olympiads. This explains why in 1978 and in 1980 there were no Olympiads (no non-socialist country was ready to organize the competition without a prior, necessary long-time preparation effort). The first IPhO organized by non-socialist country was the XIII IPhO that took place in Malente, FRG, in 1982. Then for the first time the participants solved two experimental problems (instead of one). In 1983 the IPhO was organized, for the second time, in Bucharest, Romania. Here the number of problems prepared by the organizers for the pupils much exceeded the number of problems mentioned in the Statutes and the International Board spent a lot of time discussing the Statutes and the Syllabus and the future of the Olympiads. As regards the future of the International Physics Olympiads, there was only one important decision made in Bucharest. It was decided that the next competition would take place in Sweden in 1984. Unfortunately, there were no volunteers to organize the Olympiads in 1985, 1986 and 1987. In such a situation, upon suggestion of Dr. Gunter Lind (FRG), the International Board decided to establish a permanent Secretariat (consisting of one person) for ,coordination of the long term work of the International
87
Physics Olympiads and for popularizing the Olympiads. At the same time it was decided that the Secretariat together with Prof. Lars Silverberg (Sweden), the organizer of the next competition in Sigtuna in 1984, should prepare a new version of the Statutes. The project of the Statutes was completed and the new Statutes were accepted at the XV IPhO. There are, in fact, only minor differences between the old and new versions. The most essential difference is that the new version legalize the existence of the Secretariat of the International Physics Olympiads, consisting of two persons. Another change is that at the experimental part of the competition the participants can get one or two experimental tasks (earlier only one was allowed). One can say that the new version differs from the old one primarily in wording, which in the new version is much more precise. The delegation heads (supervisors; two persons from each participating country) form the, so called, International Board, which is the highest authority of the International Physics Olympiads. The International Board does not change significantly from year to year. The majority of members know each other very well. In the International Board there is a very pleasant, friendly atmosphere. Thanks to this attitude and good will many difficult problems can be solved without great effort. This is why the Secretariat was able, for instance, to solve the problem of organization of the International Physics Olympiads in 1985, 1986 and 1987. In 1985 the International Physics Olympiad took place in Portoro2 (Yugoslavia), in 1986 - in London-Harrow (Great Britain) and in 1987 - in Jena (GDR).
88
Here we would like to emphasize that the United Kingdom organized the XVII IPhO in London-Harrow within only two years from its entry into the competition! It was made possible through hard work, great personal dedication and great enthusiasm of Dr. Cyril Isenberg, Dr. Guy Bagnall and Mr. William Jarvis. Due to joint efforts of the Secretariat and the organizers of the competitions in 1985 (Prof. Anton Moljk and Dr. Bojan Golli) and in 1986 (Dr. Dr. Cyril Isenberg and Guy Bagnall) a new version of the Syllabus was produced. Its theoretical part was accepted in Portoroft in 1985 and the practical one in London-Harrow in 1986. Quite recently, following a suggestion of the International Board, the Secretariat prepared a new, so called, "column version" of the Syllabus. This version shows not only the "breadth" of the physics contents but also the "depth" of approach required. The Syllabus of the International Physics Nevertheless, the Olympiads is indeed very modern. International Board is always ready to introduce improvements into the Statutes and Syllabus and does this if necessary. Here we would like to point out that functioning of the Secretariat is so efficient owing to not only to personal efforts of its members but also to the kind assistance of the members of the International Board. It is proper to mention here the help of Dr. Gunter Lind (FRG), Prof. Helmuth Mayr (Austria), Prof. Lars Silverberg (Sweden), Mr. Nicola Velchev (Bulgaria) and many, many others. It is obvious that the existence of the International Physics Olympiads itself is a result of certain international co-operation. More important is a long term
89
international co-operation between the members of the International Board. This kind of co-operation exists since the very beginning, i.e. since the first IPhO. The members of the International Board exchange physics problems, books, journals, articles, they discuss their experience gained during organization of the national physics competitions etc., etc. Due to such permanent, or semi-permanent, contacts and due to existence of the International Physics Olympiads some countries have organized national physics olympiads or, at least, smaller scale competitions for selecting the teams to attend the international competition. Many countries have improved their national syllabuses on physics by introducing new approaches (e.g. in thermodynamics), new topics (e.g. relativity, quantum physics), or by reducing some parts of too traditional character (e.g. geometric optics). The significance of the International Physics Olympiads is continually increasing. The role of the International Physics Olympiads is recognized also by such international organizations as UNESCO and the EPS (European Physical Society). The first contacts with UNESCO took place way back, in 1968, but more extensive co-operation began in 1984. In the period 1984 - 1988 UNESCO supported financially the publication of the proceedings of the of the subsequent Olympiads. The proceedings were distributed to all the countries-members of UNESCO. It gave us favorable publicity. In addition, UNESCO has published several books on the physics Olympiads in various languages. Quite recently, in March 1989 UNESCO organized a meeting in Holland on the "Future Development on Mathematics and Science Olympiads". The conclusions of this meeting are included in this issue.
90 The help of UNESCO is very valuable. One should realize, however, that the purposes of UNESCO are not identical with the purposes of the International Physics Olympiads (although many points are common). For example, too quick, artificially forced, increase in the number of participating countries can cause very serious organizational (mainly financial) problems. Simply, the International Physics Olympiad should be self-governing organization and its policy should be decided by the International Board only. The co-operation with the EPS is slightly different as ' the financial possibilities of the EPS are not too great. The EPS gives us very strong moral support as well as favorable publicity and propagates our achievements among the countries-members of the EPS. It was the EPS that inspired us in preparation and publication of the booklet entitled "Procedures for Selecting Teams to the International Physics Olympiads". The booklet comprises a compilation of reports of different delegations and is very important and helpful for the countries wishing to join the competition. The booklet was prepared by the Secretariat together with Prof. Lars Silverberg and published by him privately in Lund (Sweden). In 1989 the EPS created a permanent special prize for the winner of the Olympiad for reaching the best equilibrium between the theoretical and experimental parts of the competition. The Tables I and II present some data concerning the past and future International Physics Olympiads.
91 2. COMPETITION The competition lasts for two days. The first day is devoted to theory (three problems involving at least four areas of physics taught in secondary schools). Another day is devoted to experiment (one or two problems). These two days are separated by at least one day of rest. The time allotted for solving the problems is five hours (on both occasions). Each team consists of students from general or technical secondary schools (not colleges or universities). Typically each team consists of five students (pupils) and two supervisors. The latter form the International Board. For further details concerning the teams see the Statutes of the International Physics Olympiads included in this issue. The competition problems are prepared by the organizers of the Olympiad, i.e. by the host country. The texts of the problems and their solutions are analyzed in detail during the sessions of the International Board. The International Board accepts the final wording of the problems in two working languages, i.e. in English and Russian. The texts of the German and French versions are also provided, but the wording in these languages is not analyzed. In addition, the International Board suggests an evaluation guideline or accepts suggestions of the organizers concerning this matter. The theoretical and experimental problems are discussed separately, on the day prior to each part of the competition. When the final written versions of the problems are ready, the members of the International Board translate them into mother tongue(s) of the students. Thus, each student receives the problems in his/her language (and writes the
92 solutions likewise). The assessment of the pupils' entries is performed by the Organizing Committee, with the help of local interpreters, in accordance with the general evaluation guidelines accepted earlier by the International Board. Later the grading is agreed with the delegation leaders (supervisors) of each team. The final results are accepted by the International Board. The maximum number of points available for each problem is allocated by the organizers, but the ratio of the total number of points for the theoretical problems to the total number of points for the experimental problem(s) should be 3 : 2. In practice, the maximum number of points available for each theoretical problem is 10 and for each experimental problem 10 or 20 (10 when there are two experimental problems, 20 when there is only one experimental problem). The winners of the competition receive diplomas/medals or honorable mentions according to the following rules: The mean value of points accumulated by the three best participants at the given contest is considered as 100%. The contestants who obtain more than 90% of the above mentioned mean value receive first prizes. The contestants who obtain between 78% and 90% receive second prizes. The contestants who obtain between 65% and 78% receive third prizes. The contestants who obtained between 50% and 65% receive honorable mentions. All other participants receive certificates of participation in the competition. The participant with the highest score (Absolute Winner) receives an additional prize. Some special prizes can also be awarded.
93 Table I
COUNTRIES PARTICIPATING IN THE INTERNATIONAL PHYSICS OLYMPIADS Olympiad
Year
1. Australia 2. Austria 3. Belgium 4. Bulgaria 5. Canada 6. China 7. Colombia 8. Cuba 9. Cyprus 10. Czechoslovakia 11. Denmark 12. Finland 13. France 14. FRG 15. GDR 16. Great Britain 17. Greece 18. Hungary 19. Iceland 20. Iran 21. Italy 22. Kuwait 23. Lithuanian SSR 24. Netherlands 25. Norway 26. Poland 27. Romania 28. Singapore 29. Soviet Union 30. Spain 31. Sweden 32. Thailand 33. Turkey 34. UAE 35. USA 36. Vietnam 37. Yugoslavia UNESCO EPS
1... 1 2 3 4 5 6 7 8 9 0 1 2 3
4
5 6 7 8 9 0
196...197... 198... 7 8 9 0 1 24 5 6 7 9 1 2 3 4 5 6 7 8 9
# # # # H # # # # # #
# # # H # # # # # # H # 0 # # # # # # # # # # # - # # # # # # H # # #
# H # # # # # # H # #
0# # # # # # # # H 0 # H # # # # I # # 0# # # # 00# # # 0 # # # # # # # # # # # # # ## # 0 ## # # # # # # # # # # H # # # # # # # # # # # # H # 0 # # H # # # - - - - 0 # # # # # # # # # # # # # 0 # # - - 0 - # # 0# #
# # # # W - # # H # # # # # # H # # # # # # # # 0 0 - - # # # ## # H # # # #
# # # # H # # # # # H # # # # # # # # # # # # # H # # # # # # # H #
# # # #
# # # #
# # # #
# # # -
- # # # 0
0 # # # # # # # # - # # # # ## # # H # # #
# # # # # # # # # # # # # # 0 # # # # # # # H # # # 0 # 0 # 0 # #
- - - - 0 00 0 0 0 0 00 0
94 Exptanatton to the TabLe I:
# H W U
: ; : :
- : no participation participation 0 ; observer host country willingness to start from next year declared. unofficial participiation (guest of the organizers)
Table II
OF THE ORGANIZERS OLYMPIADS INTERNATIONAL PHYSICS Past: I II III IV V VI VII VIII IX X XI XII XIII XIV XV XVI XVII XVIII XIX XX
1967 1968 1969 1970 1971 1972 1974 1975 1976 1977 1979 1981 1982 1983 1984 1985 1986 1987 1988 1989
XXI XXII XXIII XXIV XXV XXVI XXVII
1990 1991 1992 1993 1994 1995 1996
Warsaw (Poland) Budapest (Hungary) Brno (Czechoslovakia) Moscow (Soviet Union) Sofia (Bulgaria) Bucharest (Romania) Warsaw (Poland) Guestrow (GDR) Budapest (Hungary) Hradec Kralove (Czechoslovakia) Moscow (Soviet Union) Varna (Bulgaria) Malente (FRG) Bucharest (Romania) Sigtuna (Sweden) Portorot (Yugoslavia) London-Harrow (United Kingdom) Jena (GDR) Bad Ischl (Austria) Warsaw (Poland)
Future:
Groningen (The Netherlands) Havana (Cuba) Helsinki (Finland) ? ? ? ?
(USA) - not confirmed (China) - not confirmed (Australia) - not confirmed (Norway)
95 We would like to underline that the number of the first prizes is not limited. The same refers to each other category of prizes. Therefore, the increase in the score of some participants (following, for example, a discussion between the supervisors and the markers) from the group of second prize winners to the group of first prize winners does not change category of prize of any other participant. Thus, the delegation leaders do not compete against each other. One can ask: what about a team classification? The answer is very simple: such a classification does not exist. The IPhO is a competition between individuals and the Statutes establish no way of defining a team result. Nevertheless, some people try to establish a kind of unofficial team classification. Some of them take a direct sum of scores as the result of the team. Some of them take the sum of scores of the three best participants in each team. Some of them take, for each team, the tree best results in each problem independently and so on, and so on. Of course, the final table depends on the way used when calculating the team results, and probably you can always find some strange system of counting the team result that will show your team to be the best or, at least, one of the best. The assessment and its verification takes several days. Throughout this time the organizers and the International Board are hard pressed. The pupils. however, are free and this is a very pleasant time for them, real rest with various cultural events. plays, excursions, etc. At the end of the competition the results are announced
96 at an official closing ceremony at which representatives of the Ministry of Education, representatives of various scientific institutions, famous physicists, etc. are present.
3. PERSPECTIVES The financial principles of the organization of the competition are the following: *** the country which sends the team covers the return travel costs (to and from the place of the competition) of , the pupils and the accompanying persons; *** from the moment of arrival until the moment of departure all the costs are covered by the organizing country. In particular, this concerns the costs of local travels, lodging, excursions, awards, etc. The number of participating countries is continually increasing. As a result, the organization of the competition becomes more and more expensive. Moreover. it is more and more difficult to organize the experimental part of the competition so that all the students have the same experimental conditions of work. We can ask, what will be the maximum number of countries? How long can the number of participating countries increase without any perturbations (assuming the same structure of the competition)? Should we start thinking about "Olympic Villages"? Until now the the organizational of participants. countries present
organizers were always able to solve all problems related to the increasing number In our opinion the maximum number of at a given Olympiad should not exceed
97
sixty. Sixty countries times five students from each country makes 300 experimental stands. This is a very great number. Some countries, however, are able to provide such a number of identical experimental stands. Other countries can organize the experimental problem in two groups. It seems that no country will be able to organize the competition (in its present form) for more than sixty countries. Can this number be reached? Theoretically, yes. But practically, probably not. The travel expenses can limit the number of participants. Many countries may not be able to send their teams to the competition every year. The number of participating countries will probably oscillate around fifty, depending on where the organizing country is situated. This will not require "Olympic Villages". The financial situation of the of the organizers depends on the internal situation in the organizing country. In some countries all or almost all the costs are paid by the Ministry of Education. In such case the financial situation of the organizers is easy, much easier than in the case of countries in which the IPhO is treated as a private event. In the last case the budget consists of funds received from many different companies and depends very strongly on the general activity of the Organizing Committee. Rich companies are always able to support the organizers, but additional effort is required from the people involved in the organization of the competition. In our opinion, it is not necessary to change anything in the structure of the International Physics Olympiads, at least at present when the number of participating countries is 30. We believe that even in the future all the financial and organizational difficulties will be successfully solved
98 and no artificial barriers should be set up against the free increase of the number of participants. Another important point, concerning the future, refers to the competition problems, their difficulty, creative capability necessary to solve them, etc. It seems that the problems given at several recent Olympiads were not as challenging as the problems presented years earlier. The creative capability necessary for solving is declining. They are too difficult, the numerical and mathematical calculations are too complicated. It is not easy to explain this decline. Probably, the most important reason is that the number of participants is increasing. As a consequence, the organizers try to formulate the problems so that the marking is as simple as possible. For example, sometimes the exact way of solving is indicated in the text of the problem and the pupils should merely follow it step by step without too great great mental effort. Second example: solutions to some problems are associated with tedious mathematical calculations, alien to the true nature of physics, with strong chances of errors. Another reason is, probably, that the organizers try to include too many ideas and concepts in the texts of the problems, e.g. too many modern developments of physics, too many national achievements and so on. As a result, the texts of the problems are very long. Sometimes it is impossible to grasp the essence of the problem in a reasonable time. The next point concerns the computer problems. One should realize that the computer problems, especially when a computer company is one of the main sponsors. cannot be avoided. However, we should always remember that the computer in only a kind of instrument. Nothing more! The
99 computer experiments, for are example, never real experiments. The Brownian motion observed trough a microscope is a real phenomenon. But the Brownian motion produced by the computer and observed on the screen is not it is a simulation of a real phenomenon only. We are in favour of using computers at the Olympiad, but we should be careful about their applications. Computers should not dominate the physics contents of the problem. Moreover, we must not allow the students from one country to have an advantage over those from other countries. competition The above critical remarks about the problems of the several recent Olympiads stem from comparison between the earlier and the more recent years. In order to avoid any misunderstanding, we would emphasize here that in an absolute scale both the earlier and the later competition problems are of a high standard, they are stimulating and require great creativity from the participants. Some criticism, however, is necessary. It can be useful in future improvements of our Olympiads. The last point we would like to mention here is the problem of translations. This problem has several aspects. The first of them concerns various printed materials prepared by the Organizing Committee in a written form. Usually this work is carried out very well and no improvements are necessary. The next aspect concerns the oral translations at the sessions of the International Board. As it was mentioned earlier, there are only two working languages: English and Russian. Thus, there is only one translation channel: English - Russian. We should say that, in general, the translations at the sessions are riot satisfactory and must
100
be improved on. It is not easy. however, to find an interpreter combining good knowledge of both the languages, good knowledge of physics and ability for simultaneous translation from English into Russian and vice versa. Moreover, it is very expensive. Any help from international organizations in this connection would be gratefully accepted by the organizers. Unfortunately, by now no international organization, including UNESCO, showed signs of willingness to help us on this matter. The subsequent aspect concerns the translation of the pupils' solutions to the problems. The marking of the solutions is performed by the Organizing Committee which is responsible for correct translation. In the case of languages spoken by a number of countries, such as German or Spanish, there are no serious difficulties. Also there are no difficulties in the case of nations or countries with a great diaspora (e.g. Poland). But in the case of certain fringe languages the organizers sometimes face great problems. Fortunately, all the possible mistakes made during the marking procedure can be corrected at the verification sessions with the participation of the delegation leaders, although sometimes it is time consuming. The last aspect refers to the pupils' interpreters to and from their mother tongues during excursions, plays and various social events. This aspect is of less importance as all the meetings organized for the pupils have a very relaxed, friendly atmosphere and the quality of translations is not emphasized. It may even be said, although it sounds a little odd, that bad translation is sometimes desirable since then the participants try to speak in foreign languages. Such a linguistic training is necessary and useful for everybody.
101
4. LITERATURE [1] Nicolaus Vermes. International Physics Competitions 1967 - 1977, Roland Edtvds Physical Society, Budapest 1978, (first edition in Hungarian, second edition in English) [2] K. K. Kudawa, International Physics Olympiads, Ganatleba, Tbilisi 1983 (in Georgian) (3] Proceedings of the 15. International Physics Olympiad, Sigtuna, Sweden, AVC, Lund 1984 (sponsored by UNESCO) [4] R. Kunfalvi, Collection of Competition Tasks from the I
through XV International Physics Olympiads 1967 - 1984. Roland Edtvds Physical Society and UNESCO, Budapest 1985 (5] 0. F. Kabardin, V. A. Orlov, International Physics Olympiads for Pupils (in Russian), Nauka, Moskva 1985
International Physics Olympiad, Portoro2, [6] 16th Yugoslavia, ed. by A. Moljk and B. Golli, Society of Mathematicians, Physicists and Astronomers of Slovenia, Ljubljana 1985 (sponsored by UNESCO) [7] Gunter Lind, Physikalische Olympiade-Aufgaben, Aulis Verlag Deubner & Co., Kdln 1986 (in German) (8] 17th International Physics Olympiad, Harrow-London, England, Organizing Committee of the 17th IPhO, Harrow 1986 (sponsored by UNESCO) [9] 18th International Physics Olympiad Report, Ministry of Education of the GDR and Organizing Committee of the 18th IPhO, Eggersdorf 1987 (sponsored by UNESCO) [101 Collection of Competition Tasks from the I through XVII International Physics Olympiads (in Chinese), published in China under aegis of UNESCO, Beijing 1988
Olympiades de Concours des de Internationales de Physique (1967 - 1984) et
[11] Recueil des Sujets
102
Mathematique (1978 -1985), D. CROS - C.I.F.E.C.-UNESCO, Paris 1988
[12]Procedures for Selecting Teams to the International Physics Olympiads (a compilation of reports from A. different delegations), ed. by W. Gorzkowski, Kotlicki, L. Silverberg, published by L. Silverberg, Lund 19886.
[13]Olimpiadas Internacionales de Fisica 1967 - 1986, UNESCO and Oficina Regional de Educacion para America Latina y El Caribe, Santiago de Chile 1988 [14]19th International Physics Olympiad - Report, Federal Ministry of Education, Arts and Sports of the Republic of Austria, Vienna 1988 (sponsored by UNESCO)
[15]Olimpiadas Internacionales de Fisica - I a XV (1967 1984), translated by Teresa Martin Sanchez and Manuela Martin Sanchez, Instituto de Ciencias de la Educacion and Ediciones Universidad de Salamanca, Salamanca 1989 by W. ed. [16]XX International Physics Olympiad, Gorzkowski, World Scientific Publishing Company, Singapore 1990 [17]International Physics Olympiads, vol. I (collection of national reports), ed. by W. Gorzkowski, World Scientific Publishing Company, Singapore - in print [18]W. Gorzkowski, A. Kotlicki, Midzynarodowe olimpiady fizyczne, WSiP, Warszawa - in print (in Polish)
ORGANIZERS OF THE INTERNATIONAL PHYSICS OLYMPIADS
Past:
I
1967
Warsaw (Poland)
II
1968
Budapest (Hungary)
III IV
1969
Brno (Czechoslovakia)
1970
Moscow (Soviet Union)
V
1971
Sofia (Bulgaria)
VI
1972
Bucharest (Romania)
VII VIII
1974
Warsaw (Poland) Guestrow (GDR)
IX
1975 1976
X
1977
Hradec Kralove (Czechoslovakia)
XI
1979
Moscow (Soviet Union) Varna (Bulgaria) Malente (FRG)
Budapest (Hungary)
XII
1981
XIII
1982
XIV
1983
Bucharest (Romania)
XV
1984
XVI
1985
Sigtuna (Sweden) Portoroz (Yugoslavia)
XVII XVIII
1986 1987
London-Harrow (United Kingdom) Jena (GDR)
XIX
1988
XX
1989
Bad Ischl (Austria) Warsaw (Poland)
Future:
XXI
1990
Groningen (The Netherlands)
XXII
1991
Havana (Cuba)
XXIII XXIV
1992 1993
(USA) — not confirmed
XXV
1994
(China) — not confirmed
XXVI
1995 1996
(Australia) — not confirmed
XXVII
Helsinki (Finland)
(Norway) — not confirmed